Download as pdf or txt
Download as pdf or txt
You are on page 1of 49

TOPNOTCH MEDICAL BOARD PREP BIOCHEMISTRY PEARLS DIGITAL HANDOUT BY DRS.

RUBIO AND BATICULON


For inquiries visit www.topnotchboardprep.com.ph or https://www.facebook.com/topnotchmedicalboardprep/
This handout is only valid for the March 2023 PLE batch. This will be rendered obsolete for the next batch since we update our handouts regularly.

METHODS OF HEAT TRANSFER


IMPORTANT LEGAL INFORMATION • Refers on how heat is transferred from the source to another
object
The handouts, videos and other review materials, provided by Topnotch Medical Board CONDUCTION CONVECTION RADIATION
Preparation Incorporated are duly protected by RA 8293 otherwise known as the
Intellectual Property Code of the Philippines, and shall only be for the sole use of the person:
Heat transfer by
a) whose name appear on the handout or review material, b) person subscribed to Topnotch which heat is
Medical Board Preparation Incorporated Program or c) is the recipient of this electronic Heat transfer by transmitted
communication. No part of the handout, video or other review material may be reproduced,
shared, sold and distributed through any printed form, audio or video recording, electronic movement of fluid without any
medium or machine-readable form, in whole or in part without the written consent of Heat transfer – air or water – medium
Topnotch Medical Board Preparation Incorporated. Any violation and or infringement,
whether intended or otherwise shall be subject to legal action and prosecution to the full
between objects when the heated
extent guaranteed by law. via direct contact fluid is caused to Radiation travels

move away from via
DISCLOSURE the heat source Electromagnetic
The handouts/review materials must be treated with utmost confidentiality. It shall be the radiation
responsibility of the person, whose name appears therein, that the handouts/review
materials are not photocopied or in any way reproduced, shared or lent to any person or
disposed in any manner. Any handout/review material found in the possession of another
person whose name does not appear therein shall be prima facie evidence of violation of RA THERMOREGULATION IN HUMANS
8293. Topnotch review materials are updated every six (6) months based on the current
trends and feedback. Please buy all recommended review books and other materials listed Organ responsible for Hypothalamus
below. controlling body temperature,
THIS HANDOUT IS NOT FOR SALE!

serving as the body’s
thermostat
INSTRUCTIONS Four avenues of heat loss in Convection, Conduction,
To scan QR codes on iPhone and iPad
1. Launch the Camera app on your IOS device humans Radiation, Evaporation
2. Point it at the QR code you want to scan
3. Look for the notification banner at the top HEAT LOSS IN HUMANS
of the screen and tap
To scan QR codes on Android
1. Install QR code reader from Play Store
If skin temperature > • Radiation (major)
2. Launch QR code app on your device environment temperature • Conduction
3. Point it at the QR code you want to scan
4. Tap browse website If environment temperature > • Evaporation (major)
skin temperature • Convection
This handout is only valid for the March 2023 PLE batch.
This will be rendered obsolete for the next batch May affect thermoregulation
since we update our handouts regularly. by limiting sweat evaporation Humidity
and thus heat loss
BIOCHEMISTRY – PEARLS
HEAT GAIN IN HUMANS
by Shivering (has 2
Frinz Moey C. Rubio, MD Major mechanism of the types of shivering –
Ronnie E. Baticulon, MD, FAFN hypothalamus in generating heat low-intensity, high-
intensity)
PART 1 Form of shivering which primarily
uses fats (long-term use)
Low-intensity
shivering
REVIEW OF CHEMISTRY CONCEPTS Form of shivering which primarily High-intensity
LAWS OF THERMODYNAMICS uses glucose (short-term use) shivering
If two systems are both in thermal Non-shivering thermogenesis is
Brown adipocytes
equilibrium with a third system, then produced by these cells
Zeroth Law
they are in thermal equilibrium with
each other (A = C; B = C → A = B) STRUCTURE OF AMINO ACIDS AND PROTEINS
Energy can be transformed from one
AMINO ACIDS ABBREVIATED BASED ON PHONETICALLY
form to another, but can be neither
First Law SOUNDING LETTERS
created nor destroyed (Energy in a
3-LETTER 1-LETTER
system is constant)
ABBREVIATION ABBREVIATION
Existence of entropy or “disorder” in a
Second Law Arginine Arg R (aRginine)
system
Asparagine Asn N (asparagiNe)
Entropy approaches a constant value
Aspartic acid Asp D (asparDic acid)
as temperature approaches absolute Third Law
zero Glutamic acid Glu E (glutEmic acid)
Glutamine Gln Q (Qtamine)
THERMODYNAMIC PROCESS Phenylalanine Phe F (Fenylalanine)
Tyrosine Tyr Y (tYrosine)
• Refers to the energetic evolution of a thermodynamic system
Tryptophan Trp W (tWiptophan)
proceeding from an initial state to a final state (described by
process quantities) K (letter close to
Lysine Lys
L)
Occurs without loss or gain of
Adiabatic process

energy by heat GLYCINE


Isenthalpic process Occurs at constant enthalpy • Has the smallest side chain
Isentropic process Occurs at constant entropy • Only achiral amino acid
Isobaric process Occurs at constant pressure Major functions • Precursor amino acid of heme
Isochoric (isometric, synthesis
isovolumetric) Occurs at constant volume • Major inhibitory NT of the spinal
process cord (released by Renshaw cells)
Isothermal process Occurs at constant temperature
Occurs without a change in the
internal energy Tetanospasmin
Steady-state process All parameters are held constant Prevents release of glycine from Renshaw cells → spastic
(heat, enthalpy, pressure, volume, paralysis
etc.)
TOPNOTCH MEDICAL BOARD PREP BIOCHEMISTRY PEARLS DIGITAL HANDOUT BY DRS. RUBIO AND BATICULON Page 1 of 49
For inquiries visit www.topnotchboardprep.com.ph or email us at topnotchmedicalboardprep@gmail.com
This handout is only valid for the March 2023 PLE batch. This will be rendered obsolete for the next batch since we update our handouts regularly.
TOPNOTCH MEDICAL BOARD PREP BIOCHEMISTRY PEARLS DIGITAL HANDOUT BY DRS. RUBIO AND BATICULON
For inquiries visit www.topnotchboardprep.com.ph or https://www.facebook.com/topnotchmedicalboardprep/
This handout is only valid for the March 2023 PLE batch. This will be rendered obsolete for the next batch since we update our handouts regularly.
ALANINE CYSTEINE
Carrier of ammonia from the • Contains a sulfhydryl group
skeletal muscles to liver • Cystine
(site of ammonia disposal) o Two cysteines connected by a disulfide bond
• Keratin
o Contains a lot of cysteine;
VALINE, LEUCINE, ISOLEUCINE o People with curly hair have more cystine → disruption of
• Branched-chain amino acids whose metabolites accumulate in disulfide links → straightened hair
maple syrup urine disease
ASPARTATE, GLUTAMATE
• Branched-chain alpha-keto acid dehydrogenase
o Deficient enzyme • Acidic amino acids
• GABA, glutathione
PHENYLALANINE o Derivatives of glutamate
• Accumulates in phenylketonuria • Glutamate
• Precursor of tyrosine o Major excitatory NT of the CNS (brain and spinal cord)
• Phenylalanine hydroxylase • GABA
o Deficient enzyme o Major inhibitory NT of the brain
• Glycine
o Major inhibitory NT of the spinal cord

HISTIDINE, ARGININE, LYSINE
• Basic amino acids
Pare, True Love Does Not Exist
• Histamine
TRYPTOPHAN o Derivative of histidine
• Creatinine, Urea, Nitric Oxide
• Has the largest side chain
o Derivatives of arginine
• Precursor for niacin,
serotonin, and • Carnitine
melatonin o Derivative of lysine
o Allows transfer of LCFA to mitochondria for beta-
TRYp Mo Siya ‘No? oxidation

METHIONINE UNIQUE PROPERTIES OF SELECTED AMINO ACIDS
• Source of methyl groups in metabolism:
Parameter Amino Acid
o Involved in transfer of methyl groups as S-adenosylmethionine
(SAM) Valine
• Precursor of homocysteine and cysteine Branched-chain AAs Leucine
• Homocysteine Isoleucine
o Increases the risk of cardiovascular diseases Disrupts the alpha-helix structure of Proline
proteins Glycine
PROLINE Smallest side Glycine
Largest side chain Tryptophan
• An imino acid
Most basic AA Arginine
• Interrupts alpha-helices in
Most acidic AA Aspartic acid
globular proteins
Known as the 21st amino acid Selenocysteine
Known as the 22nd amino acid Pyrrolysine
Amino acid uniquely present in Domain
Pyrrolysine
Archaea
SERINE, THREONINE, TYROSINE
• Contain a polar hydroxyl group DERIVATIVE OF AMINO ACIDS
• Site for O-linked glycosylation of proteins
• Thyroxine, Melanin
o Derivatives of Tyrosine



ASPARAGINE
• Serves as site of N-
glycosylation of
proteins

Lysine Carnitine
GLUTAMINE Homocysteine
Methionine
• Major carrier of nitrogen from peripheral tissue to the liver Cysteine
(site of ammonia disposal)




TOPNOTCH MEDICAL BOARD PREP BIOCHEMISTRY PEARLS DIGITAL HANDOUT BY DRS. RUBIO AND BATICULON Page 2 of 49
For inquiries visit www.topnotchboardprep.com.ph or email us at topnotchmedicalboardprep@gmail.com
This handout is only valid for the March 2023 PLE batch. This will be rendered obsolete for the next batch since we update our handouts regularly.
TOPNOTCH MEDICAL BOARD PREP BIOCHEMISTRY PEARLS DIGITAL HANDOUT BY DRS. RUBIO AND BATICULON
For inquiries visit www.topnotchboardprep.com.ph or https://www.facebook.com/topnotchmedicalboardprep/
This handout is only valid for the March 2023 PLE batch. This will be rendered obsolete for the next batch since we update our handouts regularly.
AMINO ACIDS BASED ON NUTRITIONAL REQUIREMENT TERTIARY STRUCTURE
Essential Semiessential Nonessential • Overall, 3-dimensional shape of the protein
Amino Acids Amino Acid Amino Acids • Stabilized by disulfide bonds, hydrophobic interactions,
hydrogen bonds, and ionic interactions
PVT TIM HaLL
Always ARGues Et al
Never TYRes
Phenylalanine
Valine
Tryptophan
Threonine
All other amino
Isoleucine Arginine
acids
Methionine
Histidine
Leucine
Lysine

STRUCTURAL ORGANIZATION OF PROTEINS
• Primary structure
• Secondary structure
• Tertiary structure
• Quaternary structure

PRIMARY STRUCTURE

• Determine by the AA sequence of a protein; QUATERNARY STRUCTURE
• Reading of AA sequence: N-terminus → C-terminus
• Interaction of at least 2 peptide chains (tertiary structures)


Only protein structure NOT
Primary structure
destroyed by denaturation

SECONDARY STRUCTURE

• Folding of segments of polypeptides into geometrically
N-TERMINUS AND C-TERMINUS
ordered units
• Stabilized by hydrogen bonds N-terminus • Contains targeting signals
• Most common secondary structure • Contains retention signals for protein
Alpha-helix • Keratin, hemoglobin sorting
• Disrupted by proline, glycine C-terminus
• May be modified post-translationally via
• AAs forming a zigzag or pleated pattern addition of a lipid anchor
Beta-sheets
• Amyloid, immunoglobulin



MYOGLOBIN VS. HEMOGLOBIN
Myoglobin Hemoglobin
Structure 1 polypeptide 4 polypeptides
O2 Bound 1 O2 only 4 O2
O2 SIGMOIDAL
HYPERBOLIC
Dissociation Shows
Shows saturation
Curve cooperativity
Main Purpose For O2 storage For O2 transport


TOPNOTCH MEDICAL BOARD PREP BIOCHEMISTRY PEARLS DIGITAL HANDOUT BY DRS. RUBIO AND BATICULON Page 3 of 49
For inquiries visit www.topnotchboardprep.com.ph or email us at topnotchmedicalboardprep@gmail.com
This handout is only valid for the March 2023 PLE batch. This will be rendered obsolete for the next batch since we update our handouts regularly.
TOPNOTCH MEDICAL BOARD PREP BIOCHEMISTRY PEARLS DIGITAL HANDOUT BY DRS. RUBIO AND BATICULON
For inquiries visit www.topnotchboardprep.com.ph or https://www.facebook.com/topnotchmedicalboardprep/
This handout is only valid for the March 2023 PLE batch. This will be rendered obsolete for the next batch since we update our handouts regularly.
Ehlers-Danlos
Type Bone, skin, fetal tissue,
syndrome
V placenta
(classic type)
Type Dystrophic
Anchoring fibrils
VII epidermolysis bullosa

STRUCTURE OF IMMUNOGLOBULINS



What are the protein
structures of the • Myoglobin: Tertiary;
following: 1) Myoglobin, • 2) Hemoglobin : Quaternary
2) Hemoglobin


CONFIGURATION OF HEMOGLOBIN
Key Points Remarks
T form R form Structure Oligomeric glycoproteins
Definition Taut form Relaxed form Site of production B cells → Plasma cells
Oxygen Determines the isotype of class
Low oxygen affinity High oxygen affinity
affinity 2 Heavy chains Alpha, Delta, Epsilon, Gamma, Mu
(+) VH, CH1, CH2, CH3
Kappa, Lambda
2 Light chains
(+) VL, CL
(+) VH, VL
Fab region (+) CL, CH1
Antigen-binding site
(+) CH2, CH3
Fc region (+) CH2, CH3
Crystallizable site

Hinge Flexible area containing sulfur bonds
HEMOGLOBIN Constant region Towards the Carboxyl end
Increase in CO2 levels lowers the pH Variable region Towards the Amino end
of the blood causing hemoglobin to Bohr effect
release O2 to the body tissues PROTEOLYTIC DIGESTION OF IMMUNOGLOBULINS
Removal of O2 from hemoglobin Term Feature
causes increase in affinity of Pa-pa-in 3 syllables – 3 parts
Haldane effect Pep-sin 2 syllables – 2 parts
hemoglobin for CO2, allowing it to be
expired into the lungs
This describes to the exchange of IMPORTANT TERMS ASSOCIATED WITH IMMUNOGLOBULINS
Chloride shift
bicarbonate and chloride across the Term Feature
(Hamburger effect)
membrane of the RBCs Part of the ANTIGEN
Epitope
participating in antigen-antibody interaction
DISORDERS OF COLLAGEN SYNTHESIS
Part of the ANTIBODY
Condition Defective Step Paratope
participating in antigen-antibody interaction
Hydroxylation step
Scurvy A macromolecule which mimics the
Hydroxylases
Osteogenesis structure of an epitope
Formation of triple helix Mimitope
imperfecta (can lead to antibody response similar to the
Copper deficiency Cross-linking step one elicited by the epitope)
(Menkes disease) Lysyl oxidase
TYPES OF IMMUNOGLOBULINS
What substances are required Vitamin C, Ferrous iron, Most produced antibody in the body IgA
in the hydroxylation step of Oxygen, Alpha- Structure of IgA in the serum Monomeric
collagen? ketoglutarate Structure of IgA in the mucosal surface
Dimeric
(after being secreted)
MAJOR TYPES OF COLLAGEN FIBERS
Type Distribution in the Body Associated Condition Serum concentration of
G > A > M > D > E (GAMDE)
Type B-1 Osteogenesis Ig in serum
I (later wound healing) imperfecta type I
Type Most produced antibody
Car-2-lage Achondrogenesis type 2 IgA
II in the body
AKA ret-3-culin fibers
Ehlers-Danlos
Type Blood vessels,
syndrome
III granulation tissue
(vascular type)
(early wound healing)

Type 4 on the floor Alport syndrome
IV Basement membrane Goodpasture syndrome

TOPNOTCH MEDICAL BOARD PREP BIOCHEMISTRY PEARLS DIGITAL HANDOUT BY DRS. RUBIO AND BATICULON Page 4 of 49
For inquiries visit www.topnotchboardprep.com.ph or email us at topnotchmedicalboardprep@gmail.com
This handout is only valid for the March 2023 PLE batch. This will be rendered obsolete for the next batch since we update our handouts regularly.
TOPNOTCH MEDICAL BOARD PREP BIOCHEMISTRY PEARLS DIGITAL HANDOUT BY DRS. RUBIO AND BATICULON
For inquiries visit www.topnotchboardprep.com.ph or https://www.facebook.com/topnotchmedicalboardprep/
This handout is only valid for the March 2023 PLE batch. This will be rendered obsolete for the next batch since we update our handouts regularly.
GLUCOSE TRANSPORTERS
Complement activation IgG, IgM
GLUT Cellular
Location
Able to pass through the placenta IgG Transporter Transport
RBCs, Brain, Cornea
Rises in helminthic infection, responsible in GLUT1
IgE Placenta
mast cell activation Beta-cells
Kidneys (2)
Surface immunoglobulins on naïve B cells IgM, IgD GLUT2
Liver (2 lobes)
Primary immune response IgM GI tract (2 letters)
Facilitated
GLUT3 Brain, Placenta diffusion
Secondary immune response IgG
Adipose tissue, Striated muscle
Has immunomodulatory function IgD GLUT4 (cardiac, skeletal)
Insulin-dependent
STRUCTURE OF CARBOHYDRATES GLUT5 Fructose (GI tract, sperms)
• Classification of CarbohydratesSugar units are linked together Secondary
via glycosidic bonds SGLT1/2 Kidneys, small intestines active
CLASSIFICATION NUMBER OF SUGAR UNITS transport
• One sugar unit
Monosaccharides
• Building block of all carbohydrates
STRUCTURE OF LIPIDS
Disaccharides • Two sugar units
CLASSIFICATION OF FATTY ACIDS
Oligosaccharides • 3 to 10 sugar units
Polysaccharides • More than 10 sugar units • Can be classified depending on length, presence of double
bonds, and number of double bonds (if present)

CLINICALLY IMPORTANT HEXOSES • Short chain FA C2-C6
Medium chain FA C8-C14
• Main metabolic fuel for brain, renal medulla,
Long chain FA At least C16
cornea, retina, testis, RBC
Very long chain FA At least C22
• Most predominant sugar in human body;
Glucose
universal fuel of fetus
• Oxidation yields glucuronic acid Long-chain
• Transported in chylomicrons
• Reduction yields sorbitol fatty acids
• Obtained from fruit juices Medium-chain • Directly absorbed in the bloodstream →
Fructose fatty acids hepatic portal vein
• Accumulates in fructose intolerance
• Obtained from lactose in dairy products • Directly absorbed in the bloodstream →
Short-chain
Galactose • Synthesized in the mammary glands hepatic portal vein
fatty acids
• Constituent of glycolipids and glycoproteins • “food” for the colonocytes
Mannose • Constituent of glycoproteins
DEPENDING ON THE PRESENCE OF DOUBLE BOND
CLINICALLY IMPORTANT PENTOSES • NO DOUBLE BOND in the hydrocarbon
• Structural component of nucleic acids and Saturated chain
coenzymes FA • FA is saturated with hydrogen!
Ribose • Means that all C-C bonds are single bonds
• Removal of the hydroxyl group at C2 yields
deoxyribose Unsaturated • AT LEAST ONE DOUBLE BOND in the
Ribulose • Intermediate in pentose phosphate pathway FA hydrocarbon chain is present
Xylulose • Excreted in the urine in essential pentosuria

CLINICALLY IMPORTANT DISACCHARIDES
Glucose + • Sucrase deficiency leads to
Sucrose
Fructose diarrhea and flatulence
• Lactase deficiency leads to
Glucose + diarrhea and flatulence
Lactose
Galactose • May be excreted in the urine in
pregnancy
Glucose + • Intermediate in the digestion of
Maltose
Glucose starch

OTHER POLYSACCHARIDES
Cellulose • Chief constituent of plant cell walls GEOMETRIC ISOMERISM OF FATTY ACIDS
Cis-form H atoms are found on the SAME SIDE
• Polysaccharide of fructose that is readily
Trans-form H atoms are found on the OPPOSITE SIDE
Inulin soluble in water and is used to determine

glomerular filtration rate
Monounsaturated
• Chief constituent of fungal cell walls ONLY 1 DOUBLE BOND is present
Chitin FA
• Part of exoskeleton of insects, crustaceans Polyunsaturated MORE THAN 1 DOUBLE BONDS are
FA present










TOPNOTCH MEDICAL BOARD PREP BIOCHEMISTRY PEARLS DIGITAL HANDOUT BY DRS. RUBIO AND BATICULON Page 5 of 49
For inquiries visit www.topnotchboardprep.com.ph or email us at topnotchmedicalboardprep@gmail.com
This handout is only valid for the March 2023 PLE batch. This will be rendered obsolete for the next batch since we update our handouts regularly.
TOPNOTCH MEDICAL BOARD PREP BIOCHEMISTRY PEARLS DIGITAL HANDOUT BY DRS. RUBIO AND BATICULON
For inquiries visit www.topnotchboardprep.com.ph or https://www.facebook.com/topnotchmedicalboardprep/
This handout is only valid for the March 2023 PLE batch. This will be rendered obsolete for the next batch since we update our handouts regularly.
ESSENTIAL FATTY ACIDS Highest TAG content Chylomicron
• Fatty acids that are ESSENTIAL in human diet Highest cholesterol content LDL
• Rationale: Humans lack the enzymes that can introduce Highest protein content HDL
double bonds beyond the 9th carbon

Linoleic acid LIPOPROTEIN SOURCE PROTEIN LIPID REMARKS


Essential FA Largest
α-Linolenic acid
diameter,
Semi-essential FA Arachidonic acid 98–
Chylomicron Intestine 1–2% Lowest density,

99%
LinoleNic acid = Not the parent compound of arachidonic acid Highest
Linoleic acid = parent compound of arachidonic acid TAG content
Dr. Rubio 90–
VLDL Liver 7–10% -

93%
• Integral components of membrane structure IDL VLDL 11% 89% -
• Needed for the synthesis of arachidonic Highest
Functions LDL VLDL 21% 79% cholesterol
acid (parent compound of the eicosanoids –
of content
prostaglandins, prostacyclin, leukotrienes, Liver 43– Highest protein
essential HDL 32–57%
thromboxane) Intestine 68% content
fatty acids
• Lower the risk of cardiovascular disease
• Lower the risk of fatty liver disease

OMEGA FATTY ACIDS
• Depending on the position of the first double bond from the
terminal methyl end, fatty acids are classified as:
• α-Linolenic acid
ω-3
• Timnodonic acid (eicosa-pentaenoic acid)
series
• Cervonic acid (docosahexaenoic acid) (DHA)
ω-6 • Linoleic acid
series • Arachidonic acid
ω-9 • Oleic acid
series • Elaidic acid

Omega Fatty Acids – Omega-3 series
• α-LinoleNic acid
• Timnodonic acid (eicosa-pentaenoic acid)
ω-3 series
• Cervonic acid (docosa-hexaenoci acid)
(DHA)
• Decreases serum triglycerides → ↓ risk of cardiovascular
disease
• Lowers the production of thromboxane → ↓ tendency for
platelet aggregation
• Linoleic acid → arachidonic acid →
Thromboxane thromboxane

• Omega-6 series
Carrier of free fatty acids in
Albumin
the blood
TRIACYLGLYCEROL

CHOLESTEROL
• Main storage form of lipids in the • Important steroid in animal tissues
body – stored in the adipose tissues 27-carbon compound
• Also known as triglyceride, neutral fat Structure Steroid nucleus: 4 fused
hydrocarbon rings (A-D)
Storage form Cholesterol ester
PLASMA LIPOPROTEINS Major route of
• Spherical macromolecular complexes of lipids + proteins elimination Bile excretion
from the body
Lipoprotein = Lipids + Apolipoproteins

• Structure:
o Core: Neutral lipids
o Shell: Amphipathic apolipoproteins, phospholipid, and free
fatty acids


DERIVATIVES
Adrenal hormones Cholesterol desmolase (side chain
cleavage cytochrome P450 enzyme)
Sex hormones (Rate-limiting step)
Derived from 7-dehydrocholesterol
Vitamin D
(skin)




TOPNOTCH MEDICAL BOARD PREP BIOCHEMISTRY PEARLS DIGITAL HANDOUT BY DRS. RUBIO AND BATICULON Page 6 of 49
For inquiries visit www.topnotchboardprep.com.ph or email us at topnotchmedicalboardprep@gmail.com
This handout is only valid for the March 2023 PLE batch. This will be rendered obsolete for the next batch since we update our handouts regularly.
TOPNOTCH MEDICAL BOARD PREP BIOCHEMISTRY PEARLS DIGITAL HANDOUT BY DRS. RUBIO AND BATICULON
For inquiries visit www.topnotchboardprep.com.ph or https://www.facebook.com/topnotchmedicalboardprep/
This handout is only valid for the March 2023 PLE batch. This will be rendered obsolete for the next batch since we update our handouts regularly.

7-alpha hydroxylase PuGA = Purines, Guanine & Adenine (are seen in DNA and RNA)
Cholesterol → 7-alpha-hydroxy CUT the PY (Pie) = Pyrimidines, Cytosine, Uracil, Thymine
Rate-limiting step Dr. Rubio
cholesterol

PENTOSE SUGAR
(rate-limiting step)
• Nucleotide = (Nitrogenous base + Pentose sugar) + Phosphate
Primary bile acids Cholic acid

Synthesized by the liver Chenodeoxycholic acid RNA D-Ribose sugar (ribose)


DNA 2’-deoxy D-Ribose sugar (deoxyribose)
Deoxycholic acid (from cholic
Secondary bile acids
acid)
From degradation in the NUCLEOSIDE
Lithocholic acid (from
colon by bacteria • Nucleotide = (Nitrogenous base + Pentose sugar) + Phosphate
chenodeoxcholic acid)
Taurine, glycine
Bile salts
(e.g. taurocholic acid, glycocholic
Conjugated to
acid etc.)

Most abundant bile acid Cholic acid
Most toxic bile acid Lithocholic acid


STRUCTURE OF NUCLEOTIDES NUCLEOTIDE
COMPONENTS OF NUCLEOTIDES • Nucleotide = (Nitrogenous base + Pentose sugar) + Phosphate
• NucleoTide = (Nitrogenous base + Pentose sugar) + PhosphaTe Anhydride bonds
• NucleoSide = Nitrogenous base + Pentose Sugar Ester bond

N-glycosidic bond


NUCLEIC ACIDS
• Are polymers of nucleotides joined by a 3’-5’ phosphodiester
bond
NITROGENOUS BASE • Deoxyribonucleic acid (DNA) and Ribonucleic acid (RNA)
• Nucleotide = (Nitrogenous base + Pentose sugar) + Phosphate • Polyfunctional acids
• Refers to the nitrogen-containing heterocyclic ring structures General
• Negatively charged at physiologic pH
• Purines & Pyrimidines Properties
• Absorbs light at 260 nm at pH of 7.0
• Adenine
Purine • Guanine Writing the base
• 5’ end → 3’ end
• Xanthine, Hypoxanthine sequence
• Cytosine
Pyrimidine • Uracil
• Thymine
TOPNOTCH MEDICAL BOARD PREP BIOCHEMISTRY PEARLS DIGITAL HANDOUT BY DRS. RUBIO AND BATICULON Page 7 of 49
For inquiries visit www.topnotchboardprep.com.ph or email us at topnotchmedicalboardprep@gmail.com
This handout is only valid for the March 2023 PLE batch. This will be rendered obsolete for the next batch since we update our handouts regularly.
TOPNOTCH MEDICAL BOARD PREP BIOCHEMISTRY PEARLS DIGITAL HANDOUT BY DRS. RUBIO AND BATICULON
For inquiries visit www.topnotchboardprep.com.ph or https://www.facebook.com/topnotchmedicalboardprep/
This handout is only valid for the March 2023 PLE batch. This will be rendered obsolete for the next batch since we update our handouts regularly.
(6) Chargaff’s Rule
• Number of purines = number of pyrimidines
• If the sample of DNA has 23% adenine, what will be the amount
of guanine present in the sample?
o 23% Adenine = 23% Thymine
o 100% - (23% A + 23% T) = 54% (G and C)
o 54% (G and C) / 2 = 27% (G or C)

TYPES OF DNA
• 6 types have been identified, but 3 are more prominent
A-DNA B-DNA Z-DNA
Number of
base pairs 11 10 12
DNA RNA per turns
Mostly seen in NUCLEUS Mostly seen in CYTOPLASM Broad and Longer and Longer and
Morphology
Pyrimidine bases: Pyrimidine base: short thinner thinner
THYMINE, CYTOSINE URACIL, CYTOSINE Screw Right-
Right-handed Left-handed
Sugar: DEOXYRIBOSE Sugar: RIBOSE sense handed
Usually DOUBLE- Found in low
SINGLE-STRANDED humidity
Most common
Seen in 5’ end of
STRANDED Features form
and high salt chromosomes
physiologically
conditions
SALIENT FEATURES OF THE WATSON-CRICK DNA MODEL

• (1) Right-handed (clockwise) double-stranded DNA helix
Protein-coding RNA mRNA
rRNA
tRNA
Nonprotein-coding RNA snRNA
miRNA/siRNA
lncRNA

Most heterogeneous RNA Messenger RNA (mRNA)
Most abundant RNA Ribosomal RNA (rRNA)
RNA contains unusual bases Transfer RNA (tRNA)
Small nuclear RNA
(snRNA), micro RNA
RNA regulating gene
(miRNA), small interfering
expression
RNA (siRNA), long
noncoding RNA (lncRNA)

NUTRITION
• (2,3) Base pairing rule and hydrogen bonding GROWTH FAILURE ASSOCIATED WITH MALNUTRITION
Wasting Stunting

o G-C bonding is stronger than A-T bonding Definition Thinness Shortness
o Determine the melting point of DNA (↑ G-C content; ↑ melting Acute Chronic
temperature) Form of malnutrition
malnutrition malnutrition
o GC couples flock AT the library!
o GC couples are STRONGER TOGETHER because they help Weight-for-
each other out to have high grades height (WFH) Height/Length-for-
Best indicator
Weight-for- age
(4) The 2 strands are ANTIPARALLEL length (WFL)

• Length – up to 2 years of age
• Height – beyond 2 years of age
• (World Health Organization)

PROTEIN-ENERGY MALNUTRITION
MARASMUS KWASHIORKOR
Caloric Protein deprivation > caloric
Cause
deprivation deprivation
• Edema (due to ↓ plasma
oncotic pressure due ↓
proteins)
• Generalized • Small child w/ swollen
Salient muscle abdomen
features wasting • Fatty liver (↓ amino acid pool
• No edema → ↓ apolipoproteins)
• Hyperpigmented,
dyspigmented skin (flaky
(5) Grooves of the DNA paint appearance)

Major groove
2 types of grooves
Minor groove
Sites of DNA-protein
Function of grooves interaction needed for gene
expression
Forces maintaining DNA- Hydrophobic interactions
protein interaction Ionic bonding
TOPNOTCH MEDICAL BOARD PREP BIOCHEMISTRY PEARLS DIGITAL HANDOUT BY DRS. RUBIO AND BATICULON Page 8 of 49
For inquiries visit www.topnotchboardprep.com.ph or email us at topnotchmedicalboardprep@gmail.com
This handout is only valid for the March 2023 PLE batch. This will be rendered obsolete for the next batch since we update our handouts regularly.
TOPNOTCH MEDICAL BOARD PREP BIOCHEMISTRY PEARLS DIGITAL HANDOUT BY DRS. RUBIO AND BATICULON
For inquiries visit www.topnotchboardprep.com.ph or https://www.facebook.com/topnotchmedicalboardprep/
This handout is only valid for the March 2023 PLE batch. This will be rendered obsolete for the next batch since we update our handouts regularly.
Parameter Feature Value
Protein
Digestibility- Based on profile
Corrected Amino of essential
Acid Score amino acids 0 to 1
(PDCAAS) 0: lowest value
Biological Value 1: highest value
See below
(BV)
Net Protein
See below
Utilization (NPU)
Determined by ≥2.7: Excellent
Protein Efficiency the efficiency of protein source
Ratio (PER) animal growth <2.7: Poor
(rats) protein source

BREASTFEEDING – HORMONES

(A) Kwashiorkor; (B) Marasmus For ductal development Estrogen
From First Aid for the USMLE 2020, 30th Edition For epithelial differentiation,
Progesterone
Kwashiorkor – results from protein-deficient MEALS lobular development
Malnutrition, Edema, Anemia, Liver (fatty), Skin lesions Stimulates lactogenesis Prolactin
Marasmus – Muscle wasting For milk expulsion Oxytocin

DIETARY ASSESSMENT – ENERGY CONTENT OF FOOD BREASTFEEDING – TYPES OF MILK PRODUCED
• Calculated from the heat released by the total combustion of • Production at the 12th – 16th week of
food in a calorimeter Colostrum pregnancy
Carbohydrates 4 kcal/g (deep • Rich in immunoglobulins (IgA)
Protein 4 kcal/g lemon- • More minerals and amino acids
yellow • Less sugar and fat
Fat 9 kcal/g
liquid) • Vitamin K (virtually absent)
Alcohol 7 kcal/g
• Vitamin D (low content)
1 kcal = 4.2 kJ
• Secreted by 4-6 weeks puerperium
• What is the total amount of kcal found in a diet consisting of 1 g
• Less minerals and amino acids
of carbohydrates, 3 g of fat, 4 g of protein and 2 g of alcohol? Mature
• More sugar and fat
o (1 x 4) + (3 x 9) + (4 x 4) + (2 x 7) milk
o = 61 kcal • Vitamin K (virtually absent)
• Vitamin D (low content)
DIETARY ASSESSMENT –
DISTRIBUTION OF TOTAL ENERGY EXPENDITURE VITAMINS & MINERALS
• Total Energy Expenditure (EER): number of kcal expended by NUTRIENTS
the abovementioned processes in a 24-hour period • Food constituents necessary to sustain the normal bodily
functions
• Classified as either a macronutrient or micronutrient
(depending on amount needed by the body)


DIETARY ASSESSMENT –
ACCEPTABLE MACRONUTRIENT DISTRIBUTION RANGES

Macronutrient Micronutrient
Needed in large amounts Needed in small amounts
Can provide calories
Proteins
Carbohydrates Vitamins
Lipids Minerals
Ethanol

Saturated fat and trans fat <10% of total kcal VITAMINS VS. MINERALS
Polyunsaturated fatty acids (PUFA) 5-10% of total kcal Vitamins Minerals
Monounsaturated fatty acids (MUFA) 10-20% of total kcal Organic substances Inorganic substances
Simple sugars <10% of total kcal Easily destroyed by cooking Not vulnerable to heat,
with heat, chemical reagents sunlight, or chemical agents
DIETARY PROTEIN QUALITY MEASUREMENT
• Protein Digestibility-Corrected Amino Acid Score (PDCAAS) Due to ACCUMULATION of a certain nutrient in
• Biological Value (BV) Toxicity
the body
• Net Protein Utilization (NPU)
• Protein Efficiency Ratio (PER) Due to LACK OF A CERTAIN NUTRIENT in the
Deficiency
body



TOPNOTCH MEDICAL BOARD PREP BIOCHEMISTRY PEARLS DIGITAL HANDOUT BY DRS. RUBIO AND BATICULON Page 9 of 49
For inquiries visit www.topnotchboardprep.com.ph or email us at topnotchmedicalboardprep@gmail.com
This handout is only valid for the March 2023 PLE batch. This will be rendered obsolete for the next batch since we update our handouts regularly.
TOPNOTCH MEDICAL BOARD PREP BIOCHEMISTRY PEARLS DIGITAL HANDOUT BY DRS. RUBIO AND BATICULON
For inquiries visit www.topnotchboardprep.com.ph or https://www.facebook.com/topnotchmedicalboardprep/
This handout is only valid for the March 2023 PLE batch. This will be rendered obsolete for the next batch since we update our handouts regularly.
FAT-SOLUBLE VS. WATER-SOLUBLE VITAMINS VITAMIN A DEFICIENCY
Fat-Soluble Vitamins Water-Soluble Vitamins • Most common vitamin deficiency worldwide (and in the
Vitamin B complex Philippines)
Vitamin ADEK • Most common cause of preventable blindness worldwide
Vitamin C
Soluble in lipid medium Soluble in aqueous medium Collective term for ALL ocular
Xerophthalmia
Absorbed via chylomicrons manifestations of Vitamin A deficiency
Directly absorbed into the Loss of sensitivity
(lymphatic system) → Earliest SIGN of vitamin A deficiency
circulation to green light
circulation
Stored in liver and adipose Circulate freely in water-filled Nyctalopia Earliest SYMPTOM of Vitamin A
tissue parts of the body (night blindness) deficiency
Not readily excreted Readily excreted in urine Xerosis Dryness of the conjunctiva
HIGH risk for toxicity LOW risk for toxicity White patches of keratinized
Bitot spots
“Kapag ADEK ang isang tao sa’yo, usually mas CLINGY siya, mas epithelium appearing on the sclera
TOXIC.” "
$
# Keratomalacia Softening of the cornea
Dr. Rubio
Corneal ulceration (painful) → corneal
Corneal ulceration

ENDOGENOUSLY SYNTHESIZED VITAMINS scarring (blindness)


Vitamin Source
• From precursor steroids Phrynoderma Epithelial metaplasia and keratinization →
• 7-dehydrocholesterol: found in or Toad Skin follicular hyperkeratosis
Vitamin D
epidermis; converted to vitamin D3 by Squamous metaplasia → loss of cilia,
RT, GIT
sunlight microvilli, and mucus production → ↑ stasis
metaplasia
Vitamin K of bacteria → ↑ risk of infection
Biotin • From intestinal microflora
Pantothenic acid Common in arctic explorers eating polar bear
Occurrence
• From tryptophan (an essential AA) – liver
Niacin
requires vitamin B2, vitamin B6 Pseudotumor cerebri: headache, vomiting,
Acute toxicity blurred vision, stupor (that may be confused
STORAGE SITES OF VITAMINS IN THE BODY w/ brain tumor)
STORAGE Chronic Increased risk for fractures (hip)
MAJOR SITE OF STORAGE PERIOD IN toxicity Teratogenic in pregnancy
THE BODY
FAT-SOLUBLE VITAMINS VITAMIN D
Stellate cells of Ito 6 months • Refers to group of sterols having a hormone-like function
Vitamin A (perisinusoidal space of the (Robbins & Ergocalciferol Form of vitamin D obtained from fungal
LIVER) as Retinyl Ester Cotran) (Vitamin D2) organisms (mushroom, ergots)
Vitamin D Liver --- Cholecalciferol From endogenous synthesis of 7-
Vitamin E Liver --- (Vitamin D3) dehydrocholesterol via UVB radiation
Vitamin K Liver ---

FUNCTIONS OF VITAMIN D
WATER-SOLUBLE VITAMINS
• Regulation of calcium and phosphorus homeostasis
Vitamin B9 3-4 months • Immunomodulation and antiproliferative effects:
Liver
Folic acid (Harrison’s) o Prevents infection by Mycobacterium tuberculosis
Vitamin B12 3-4 years o Less than 20 ng/mL of vitamin D: associated with increase in
Liver
Cobalamin (Harrison’s) incidence of colon, breast, and prostate cancer

VITAMIN A • Mineralization of bones


• In the strictest sense, vitamin refers to retinol ↑ vitamin D (active form) = ↑ Calcium, ↑Phosphorus
• Retinoids: refers to all compounds chemically related to retinol:
Retinal 11-cis-retinal: for normal vision VITAMIN D DEFICIENCY AND TOXICITY
For normal morphogenesis, growth & cell VITAMIN D DEFICIENCY VITAMIN D TOXICITY
Retinoic acid • Rickets: before closure • ↑ Vitamin D → ↑ Ca2+
differentiation
For reproduction of cells of epiphysis (children) • Blood vessel contraction →
Retinol • Osteomalacia: after hypertension
Form absorbed by intestinal cells
Retinyl ester Storage form of vitamin A in liver closure of epiphysis • Calcification of soft tissues
(adults) (metastatic calcification)
Metarhodopsin II
Activation of transducin → hyperpolarization of retina VITAMIN E
• Carotenoids: refers to provitamins of Vitamin A present in • All stereoisomers of tocopherols & tocotrienols
plants; as provitamins they can be metabolized to vitamin A • Most powerful naturally occurring antioxidant
Most prevalent carotenoid in the food α-tocopherol Most potent form of vitamin E
β-carotene
supply that has provitamin A activity
Lutein Protection against macular degeneration Micronutrients with Antioxidant Effects
Lycopene Protection against prostate cancer Vitamin A

FUNCTIONS OF VITAMIN A Vitamin C


• Rhodopsin: most light sensitive pigment Vitamin E (most potent)
Maintenance Zinc
present in rods; formed by covalent
of vision Selenium
association between 11-cis-retinal & opsin
• Retinol: supports reproduction of cells
Normal • Retinoic acid: supports embryogenesis VITAMIN E DEFICIENCY VITAMIN E TOXICITY
reproduction • Uses Retinoic acid receptors (RARs) and • Axonal degeneration →
Retinoic X receptors (RXRs) muscle weakness
• Deficiency of vitamin A can lead to • Demyelination of • Reduces platelet aggregation
Maintenance squamous metaplasia (columnar → posterior columns & • Interferes with the effects of
of skin and squamous) of respiratory and intestinal spinocerebellar tract Vitamin K
mucosa tract → ↑ risk for infections (especially • Acanthocytosis of RBCs
measles infection) → hemolytic anemia
Antioxidant • β-carotene: has antioxidant properties via
properties replenishment of vitamin E in the blood
TOPNOTCH MEDICAL BOARD PREP BIOCHEMISTRY PEARLS DIGITAL HANDOUT BY DRS. RUBIO AND BATICULON Page 10 of 49
For inquiries visit www.topnotchboardprep.com.ph or email us at topnotchmedicalboardprep@gmail.com
This handout is only valid for the March 2023 PLE batch. This will be rendered obsolete for the next batch since we update our handouts regularly.
TOPNOTCH MEDICAL BOARD PREP BIOCHEMISTRY PEARLS DIGITAL HANDOUT BY DRS. RUBIO AND BATICULON
For inquiries visit www.topnotchboardprep.com.ph or https://www.facebook.com/topnotchmedicalboardprep/
This handout is only valid for the March 2023 PLE batch. This will be rendered obsolete for the next batch since we update our handouts regularly.
VITAMIN K • Coenzyme for redox
• Koagulation vitamin Vitamin B3 reactions
• Required for the post-translational gamma-carboxylation of Niacin • Can be synthesized from
NAD, NADPH
glutamic acid: necessary step for calcium-binding to Nicotinic tryptophan (60 mg
carboxylated proteins: acid tryptophan = 1 mg niacin)
• Factors II, VII, IX, X (1972) – requires B2, B6
Coagulation factors Vitamin B5
• Protein C, Protein S • Coenzyme A component
Protein in bone • Osteocalcin Pantothenic Coenzyme A
• Fatty acid synthase
• Nephrocalcin (inhibitor of calcium acid
Protein in kidney • Coenzyme for AA
oxalate stone formation)
metabolism
Vitamin B6 Pyridoxal
Form of Vitamin K from Phylloquinone • Coenzyme for
Pyridoxine phosphate
dietary sources (Vitamin K1) glycogenolysis (via
Form of Vitamin K Menaquinone glycogen phosphorylase)
synthesized by bacterial flora (Vitamin K2) Vitamin • Coenzyme for
Enzyme-bound
Form of synthetic Vitamin K Menadione (Vitamin K3) B7/H carboxylation reactions
biotin
Water-soluble form of Biotin (methyl transfers)
Menadione (Vitamin K3) Redox reactions
Vitamin K
Vitamins B2, B3
Vitamin K Deficiency Vitamin K Toxicity ↑CHO intake
Vitamin B1
• Hemolysis →
• Bleeding diathesis hyperbilirubinemia → ↑Protein intake
kernicterus (CNS damage) Vitamins B6

Vitamin B Deficiencies
Risk of Infants for Vitamin K deficiency
Low fat stores in neonates Vitamin Deficiency
Absence of intestinal flora in neonates • Beriberi (wet, dry)
Absence of vitamin K in breastmilk • Wernicke syndrome (reversible) -
Vitamin B1
(confusion, ophthalmoplegia, ataxia)
Thiamine
VITAMIN C • Korsakoff syndrome (irreversible) - (onset
• Also known as anti-scorbutic (scurvy) factor of dementia and confabulation)
• Humans and other primates cannot endogenously synthesize Vitamin B2 • Cheilosis, corneal vascularization (2Cs)
vitamin C due to the absence of gulonolactone oxidase (in Riboflavin • Magenta tongue, seborrheic dermatitis
uronic acid pathway) • Pellagra (3Ds) - (dermatitis - Casal
Vitamin B3
necklace, dementia, diarrhea)
• Good reducing agent Niacin
• Conditions causing increased risk for vitamin
• Antioxidant Nicotinic
B3 deficiency: Hartnup disease, carcinoid
• Posttranslational modification, acid
syndrome, isoniazid use
hydroxylation of lysine & proline in collagen Vitamin B5 • Gopalan’s burning feet syndrome
synthesis Pantothenic (nutritional melalgia)
• Converts dopamine to norepinephrine (via acid • Paa-nthothenic
General
dopamine β-hydroxylase)
Functions • Neurological manifestations (due to
• Iron absorption via conversion of ferric
problems in NT metabolism)
(Fe3+) ions to ferrous (Fe2+) ions
• Microcytic hypochromic anemia (due to
• Homogentisic acid synthesis
Vitamin B6 decreased heme synthesis via ALA synthase)
• Bile acid synthesis (via 7α-hydroxylase) Pyridoxine • Sideroblastic anemia (due to iron excess in
• Conversion of folate to its active form RBCs secondary to defective heme synthesis)
• Adrenal steroid synthesis • Pellagra (due to defective niacin synthesis
from tryptophan)
Fe2+ iron (ferrous state) • Long-term antibiotic use or excessive
Iron absorbed in the duodenum ingestion of RAW WHITE EGG WHITES
Vitamin
B7/H (large amounts of AVIDIN)
VITAMIN C DEFICIENCY & TOXICITY Biotin • Avidin in egg whites AVIDLY BINDS biotin
VITAMIN C DEFICIENCY VITAMIN C TOXICITY • Causes dermatitis, enteritis, alopecia
• Scurvy: Pellagra-like syndrome
o Corkscrew hair • Nausea, vomiting, Vitamin B6 > B2 deficiency
o Swollen gums diarrhea

o Bleeding diathesis • Calcium oxalate Vitamin B Toxicities
o Poor wound healing nephrolithiasis
Vitamin Toxicities
o Perifollicular & • ↑ risk for iron toxicity
Vitamin B1
subperiosteal hemorrhages (due to increased
Thiamine
• Weakened immune dietary iron absorption)
Vitamin B2
response
Riboflavin
Vitamin C megadosage Hepatotoxicity – most serious toxic reaction
Vitamin B3
GI effects – diarrhea Podagra – form of gout affecting the 1st MTP
Niacin
joint
Nicotinic acid
VITAMIN B COMPLEX – FEATURES Facial flushing
Vitamin Active Form Features Vitamin B5
• Decarboxylation of Pantothenic
Vitamin B1 Thiamine acid
alpha-keto acids,
Thiamine pyrophosphate
branched chain AAs Vitamin B6
Sensory neuropathy
Vitamin B2 • Coenzyme for redox Pyridoxine
FAD, FMN
Riboflavin reactions Vitamin
B7/H
Biotin
TOPNOTCH MEDICAL BOARD PREP BIOCHEMISTRY PEARLS DIGITAL HANDOUT BY DRS. RUBIO AND BATICULON Page 11 of 49
For inquiries visit www.topnotchboardprep.com.ph or email us at topnotchmedicalboardprep@gmail.com
This handout is only valid for the March 2023 PLE batch. This will be rendered obsolete for the next batch since we update our handouts regularly.
TOPNOTCH MEDICAL BOARD PREP BIOCHEMISTRY PEARLS DIGITAL HANDOUT BY DRS. RUBIO AND BATICULON
For inquiries visit www.topnotchboardprep.com.ph or https://www.facebook.com/topnotchmedicalboardprep/
This handout is only valid for the March 2023 PLE batch. This will be rendered obsolete for the next batch since we update our handouts regularly.
VITAMIN B9, B12 CytoplaZmic SOD
VITAMIN B9 VITAMIN B12 Requires Copper, Zinc
Site of
Jejunum Ileum DISORDERS OF COPPER METABOLISM
absorption
Methylcobalamin MENKES
Active Tetrahydrofolic WILSON DISEASE
Deoxyadenosyl- DISEASE
form(s) acid Mode of
cobalamin X-linked recessive Autosomal recessive
Transfers one- inheritance
carbon units: Cu-transporting
Coenzyme for ATP7A ATP7B
synthesis of ATPase affected
reactions: (1)
Functions (1) methionine, Whole body Cu ↓ ↑
homocysteine →
(DNA (2) serine, Free serum Cu ↓ ↑
methionine,
synthesis) (3) purine Urinary Cu ↓ ↑
(2) methylmalonyl CoA
nucleotides, Cu is not Storage of Cu in
→ succinyl CoA
(4) thymidine Pathophysiology mobilized from parenchyma of
monophosphate intestine different organs
Storage site of Vitamins B9, B12 • Kayser-Fleischer
Liver • Kinky or steely
rings
hair
Salient features • Liver dysfunction
• Neurologic
• REVERSIBLE macrocytic, megaloblastic • Neuropsychiatric
degeneration
anemia abnormalities
• Hypersegmented neutrophils VS. Vitamin C deficiency
IRREVERSIBLE Corkscrew hair
Presentation subacute combined
No neurologic degeneration of: ZINC
symptoms dorsal columns, lateral • Serves as cofactor in the following enzymes
corticospinal tracts, Alcohol dehydrogenase Ethanol → acetaldehyde
spinocerebellar tracts Carbonic anhydrase Important in buffer system
• ↑ homocysteine Porphobilinogen Heme synthesis
• ↑ homocysteine
• Normal synthase Inhibited by lead
Laboratory • ↑ methylmalonic
methylmalonic Nonmitochondrial
parameters acid Superoxide → hydrogen peroxide
acid superoxide dismutase
• ↓ folic acid Also requires copper
• ↓ folic acid (SOD)

VS. Vitamin E Deficiency


(+) demyelination of posterior columns CytoplaZmic SOD
(+) hemolytic anemia Requires Copper, Zinc
Helminth that accumulates • Structural function (zinc fingers)
cobalamin from food rendering Diphyllobothrium latum • Stabilizes the insulin hormone structure
cobalamin unavailable for (fish tapeworm)
absorption
May be used as therapy for
Acute Promyelocytic Leukemia All-transretinoic acid
(differentiation therapy)
May reduce risk of neural tube
defects, and Acute Folic acid
Lymphoblastic Lymphoma
Duration before storage of folic
3-4 months
acid in the body is depleted
Duration before storage of
cobalamin in the body is 3-4 years
depleted

MINERALS
IRON • Absolutely required for normal spermatogenesis
(1) Oxygen-binding • Fetal growth
Function of ferrous iron (Fe2+ • Embryonic development
(2) Absorption by duodenal
state, reduced state)
enterocytes
(1) For transport in the Sperm Requirements
Function of ferric iron (Fe3+ Zinc – normal spermatogenesis
blood (transferrin)
state, oxidized state) Fructose – energy of the sperm motility
(2) For storage (ferritin)

COPPER ACRODERMATITIS ENTEROPATHICA
• Serves as cofactor in the following enzymes Mode of inheritance Autosomal recessive
Transfers electrons from
Cytochrome c oxidase ↓ zinc absorption →
cytochrome c to oxygen in ETC Pathophysiology
↓ zinc in the body
Dopamine β- Dopamine → norepinephrine Vesicular rashes
hydroxylase (extremities, face)
Ferroxidases Oxidizes iron (from ferrous form Clinical Presentation
Diarrhea
(Hephaestin) → ferric form) Alopecia
Forms cross-links in collagen &
Lysyl oxidase
elastin fibers
Tyrosinase Synthesis of melanin
Superoxide → hydrogen
Cytoplasmic superoxide
peroxide
dismutase (SOD)
Also requires zinc
TOPNOTCH MEDICAL BOARD PREP BIOCHEMISTRY PEARLS DIGITAL HANDOUT BY DRS. RUBIO AND BATICULON Page 12 of 49
For inquiries visit www.topnotchboardprep.com.ph or email us at topnotchmedicalboardprep@gmail.com
This handout is only valid for the March 2023 PLE batch. This will be rendered obsolete for the next batch since we update our handouts regularly.
TOPNOTCH MEDICAL BOARD PREP BIOCHEMISTRY PEARLS DIGITAL HANDOUT BY DRS. RUBIO AND BATICULON
For inquiries visit www.topnotchboardprep.com.ph or https://www.facebook.com/topnotchmedicalboardprep/
This handout is only valid for the March 2023 PLE batch. This will be rendered obsolete for the next batch since we update our handouts regularly.
OTHER MINERALS Glycerophospholipid Feature
Function Deficiency Toxicitiy -choline Most abundant
Cofactor for -ethanolamine Second most abundant
Manganese numerous -serine Apoptosis
enzymes Reservoir of arachidonic acid
Cofactor for Neurologic -inositol Source of ITP and DAG (secondary
Molybdenum oxidase dysfunction messengers → signal transduction)
enzymes Xanthinuria Antigenic, cross reacts with
Cardiolipin
antibodies to syphilis
Constituent of Macrocytic
Cobalt
cobalamin anemia
EICOSANOIDS
Potentiates the Impaired • Derived from the 20-carbon fatty acid, arachidonic acid
Chromium action of glucose
insulin tolerance
Constituent of Dental
Fluoride Fluorosis
bone & teeth caries
Kaschin-
Keshan
Beck
cardio-
disease:
myopathy:
Selenium Antioxidant garlic
necrosis of
breath,
cardiac
brittle
muscle
nail

Mitochondrial SOD
Requires Manganese
VS. Zinc
Stabilizes insulin structure

Iron
Vitamin deficiencies associated
Vitamin C
with microcytic anemia
Vitamin B6
Vitamin deficiencies associated Vitamin B9 GENERAL ENZYMOLOGY
with macrocytic anemia Vitamin B12 ENZYMES
• Protein catalysts: increases the velocity of the reaction (but
COMPLEX MOLECULES does not affect chemical equilibrium)
GLYCOSAMINOGLYCANS (MUCOPOLYSACCHARIDES) • Mechanism: Lowers the activation energy of a chemical
GAG SUGAR UNITS LOCATION reaction
• Skin, synovial
• N- fluid, bone,
Hyaluronic
acetylglucosamine cartilage, vitreous
acid
• Glucuronic acid humor, embryonic
tissues
• N-
Chondroitin • Cartilage, bone,
acetylgalactosamine
sulfate CNS
• Glucuronic acid
• N- • Cornea, cartilage,
Keratan
acetylglucosamine loose connective
sulfate
• Galactose tissue
• Glucosamine • Mast cells, liver,
Heparin RNA molecules with
• Iduronic acid lung, skin
catalytic activity (exception Ribozymes
• Skin, kidney to the rule)
Heparan • Glucosamine
basement CLASSES OF ENZYMES
sulfate • Glucuronic acid
membrane • Add oxygen or remove hydrogens
• N- Oxidoreductases (oxidases, dehydrogenases)
Dermatan • Skin, wide
acetylgalactosamine (Class I) • Add hydrogens (reductases) to
sulfate distribution
• Iduronic acid substrates
• Move chemical groups (glycosyl,
Most common GAG in the body Transferases
methyl, phosphoryl) from one
Chondroitin sulfate (Class II)
substrate to another
• Cleave substrate bonds using water
GLYCEROPHOSPHOLIPIDS (PHOSPHOGLYCERIDES) (adding H+ or OH-) to the cleavage
Hydrolases
products
(Class III)
• Subtypes include peptidases,
lipases, etc.
Lyases • Catalyzes cleavage of C-C, C-S, and
(Class IV) certain C-N bonds
Isomerases • Rearrange substrate molecules to
(Class V) form a different isomer
• Catalyze the formation of bonds
Ligases between C and O, S, and N coupled
(Class VI) to hydrolysis of high-energy
phosphates (e.g., ATP hydrolysis)
InSidE



TOPNOTCH MEDICAL BOARD PREP BIOCHEMISTRY PEARLS DIGITAL HANDOUT BY DRS. RUBIO AND BATICULON Page 13 of 49
For inquiries visit www.topnotchboardprep.com.ph or email us at topnotchmedicalboardprep@gmail.com
This handout is only valid for the March 2023 PLE batch. This will be rendered obsolete for the next batch since we update our handouts regularly.
TOPNOTCH MEDICAL BOARD PREP BIOCHEMISTRY PEARLS DIGITAL HANDOUT BY DRS. RUBIO AND BATICULON
For inquiries visit www.topnotchboardprep.com.ph or https://www.facebook.com/topnotchmedicalboardprep/
This handout is only valid for the March 2023 PLE batch. This will be rendered obsolete for the next batch since we update our handouts regularly.
RATE-LIMITING ENZYMES
De novo pyrimidine
Process Enzyme Carbamoyl phosphate synthetase II
synthesis
Glycolysis Phosphofructokinase-1
Gluconeogenesis Fructose-1,6-bisphosphatase De novo purine
Glutamine-PRPP amidotransferase
TCA cycle Isocitrate dehydrogenase synthesis
Glycogenesis Glycogen synthase
Glycogenolysis Glycogen phosphorylase
HMP shunt G6PD SPECIAL TOPICS
Urea cycle Carbamoyl phosphate synthetase I CELLS OF THE IMMUNE SYSTEM
Fatty acid synthesis Acetyl coA carboxylase Only lymphocyte participating in the innate
NK cells
Fatty acid oxidation Carnitine acyltransferase I immune response
Ketogenesis HMG-CoA synthase Phagocyte responsible for clearing immune
Eosinophils
Cholesterol complexes
HMG-CoA reductase Link between the innate and adaptive Dendritic
synthesis
immune response cells

COMPLEMENT SYSTEM


ALZHEIMER DISEASE
Neurotransmitter decreased in PART 2
patients with Alzheimer disease
(basis of current drugs for this
Acetylcholine
disease – cholinesterase inhibitors:
donepezil, rivastigmine,
galantamine)
Down syndrome
Congenital syndrome associated (due to early deposition
with early Alzheimer disease of extracellular amyloid
deposits)

ISOLATED INDIRECT HYPERBILIRUBINEMIA
Crigler-Najjar syndrome
Complete absence of UDPGT
type I
activity
(worst prognosis)
Partial absence of UDPGT
Crigler-Najjar syndrome
activity (remaining 10%
type II
activity)
10-35% remaining UDPGT
Gilbert syndrome
activity

ISOLATED DIRECT HYPERBILIRUBINEMIA
With defective hepatic drug
reuptake transporters - Rotor syndrome
OATP1B1, OATP1B3
With mutations in gene for
Dubin-Johnson syndrome
MRP2








TOPNOTCH MEDICAL BOARD PREP BIOCHEMISTRY PEARLS DIGITAL HANDOUT BY DRS. RUBIO AND BATICULON Page 14 of 49
For inquiries visit www.topnotchboardprep.com.ph or email us at topnotchmedicalboardprep@gmail.com
This handout is only valid for the March 2023 PLE batch. This will be rendered obsolete for the next batch since we update our handouts regularly.
TOPNOTCH MEDICAL BOARD PREP BIOCHEMISTRY PEARLS DIGITAL HANDOUT BY DRS. RUBIO AND BATICULON
For inquiries visit www.topnotchboardprep.com.ph or https://www.facebook.com/topnotchmedicalboardprep/
This handout is only valid for the March 2023 PLE batch. This will be rendered obsolete for the next batch since we update our handouts regularly.
C. Neither


High insulin: Fructose-1,6-bisphosphatase is ______________
Low glucagon: Hexokinase is _____________________
Low insulin:glucagon ratio: glycogen phosphorylase is ______
High cortisol: HMG CoA reductase is _____________

High cAMP: Hormone sensitive lipase is ______________
A. Activated
B. Inhibited
C. Neither

Glycerol for gluconeogenesis

Fatty acids for β-oxidation


Phosphorylation: Glycogen synthase is _________________
Dephosphorylation: Pyruvate dehydrogenase is _______
A. Activated
B. Inhibited
Which of the following statements is INCORRECT?
A. Glutamate is negatively charged at physiologic pH
1. B. Isoleucine has an alipathic R group
C. Tyrosine has an aromatic R group
D. The pI of Arginine is less than 7

TOPNOTCH MEDICAL BOARD PREP BIOCHEMISTRY PEARLS DIGITAL HANDOUT BY DRS. RUBIO AND BATICULON Page 15 of 49
For inquiries visit www.topnotchboardprep.com.ph or email us at topnotchmedicalboardprep@gmail.com
This handout is only valid for the March 2023 PLE batch. This will be rendered obsolete for the next batch since we update our handouts regularly.
TOPNOTCH MEDICAL BOARD PREP BIOCHEMISTRY PEARLS DIGITAL HANDOUT BY DRS. RUBIO AND BATICULON
For inquiries visit www.topnotchboardprep.com.ph or https://www.facebook.com/topnotchmedicalboardprep/
This handout is only valid for the March 2023 PLE batch. This will be rendered obsolete for the next batch since we update our handouts regularly.


Consider the following tetrapeptide:
Ala-Glu-Gly-Lys
Alanylglutamylglycyllysine

2. Which of the following statements is TRUE?

A. Edman’s reagent will remove glutamate first
B. The c-terminal AA is alanine
C. The free α-amino group is found in glycine
D. The free α-carboxyl group is found in lysine

Regarding carbon monoxide poisoning, which of the
following statements is INCORRECT?
A. In healthy individuals, 1% or less of total
hemoglobin is bound to carbon monoxide
B. COHb levels in smokers are usually between 3 to
3.
8%, and may be higher among chain smokers
C. Dizziness, nausea, and confusion begin at COHb
levels > 30%
D. Death normally occurs when COHb level is >
60%

Regarding porphyrins, which of the following statements
is INCORRECT?
A. It consists of four pyrrole rings linked together
by methyne bridges
B. The porphyrin ring may exist in either chair or
4.
boat conformation
C. In chlorophyll, porphyrin binds to magnesium
instead of iron
D. Porphyrinogens are colorless, whereas
porphyrins are colored

TOPNOTCH MEDICAL BOARD PREP BIOCHEMISTRY PEARLS DIGITAL HANDOUT BY DRS. RUBIO AND BATICULON Page 16 of 49
For inquiries visit www.topnotchboardprep.com.ph or email us at topnotchmedicalboardprep@gmail.com
This handout is only valid for the March 2023 PLE batch. This will be rendered obsolete for the next batch since we update our handouts regularly.
TOPNOTCH MEDICAL BOARD PREP BIOCHEMISTRY PEARLS DIGITAL HANDOUT BY DRS. RUBIO AND BATICULON
For inquiries visit www.topnotchboardprep.com.ph or https://www.facebook.com/topnotchmedicalboardprep/
This handout is only valid for the March 2023 PLE batch. This will be rendered obsolete for the next batch since we update our handouts regularly.


What is the correct sequence of enzymes in bilirubin
metabolism?
A. Biliverdin reductase, Bilirubin UDP-
glucuronyltransferase, Heme oxygenase
B. Bilirubin UDP-glucuronyltransferase, Biliverdin
5.
reductase, Heme oxygenase
C. Heme oxygenase, Biliverdin reductase, Bilirubin Which of the following statements describes the classical
UDP-glucuronyltransferase pathway for complement activation?
D. Heme oxygenase, Bilirubin UDP- A. C3b binds to the surface of the pathogen
glucuronyltransferase, Biliverdin reductase B. Cleavage of C3 to C3a and C3b
6.
C. IgM and/or IgG bind to antigen and form
antigen-antibody complexes
D. MBL binds to mannose-containing
polysacchardies on the surface of bacterial cells


Regarding immunoglobulins, which of the following is INCORRECT?
A. IgM has the highest molecular weight
7. B. IgA can exist either as a monomer or a dimer
C. IgG is the most abundant in serum
D. IgE can cross the placenta


TOPNOTCH MEDICAL BOARD PREP BIOCHEMISTRY PEARLS DIGITAL HANDOUT BY DRS. RUBIO AND BATICULON Page 17 of 49
For inquiries visit www.topnotchboardprep.com.ph or email us at topnotchmedicalboardprep@gmail.com
This handout is only valid for the March 2023 PLE batch. This will be rendered obsolete for the next batch since we update our handouts regularly.
TOPNOTCH MEDICAL BOARD PREP BIOCHEMISTRY PEARLS DIGITAL HANDOUT BY DRS. RUBIO AND BATICULON
For inquiries visit www.topnotchboardprep.com.ph or https://www.facebook.com/topnotchmedicalboardprep/
This handout is only valid for the March 2023 PLE batch. This will be rendered obsolete for the next batch since we update our handouts regularly.


Which immunoglobulin is most important for mucosa- Which group of patients are at risk of developing
associated lymphoid tissue? Alzheimer disease at age 40 years or younger?
A. IgA A. Down syndrome
8. 10.
B. IgD B. Klinefelter syndrome
C. IgE C. Marfan syndrome
D. IgM D. Zellweger syndrome

Which of the following is the major protein in human Which of the following statements is INCORRECT
plasma, responsible for about 75% of the oncotic regarding Alzheimer disease?
pressure in blood? A. Brain biopsy typically show evidence of
9. A. Albumin cholinergic excess
B. α1-globulin B. Extracellular plaques consist mainly of amyloid
C. β-globulin 11. β-protein
D. γ-globulin C. Individuals homozygous for APOE4 have a 16-
fold higher risk of developing AD
D. Neuronal tangles consist of microtubule-
associated protein tau, which is normally
soluble


TOPNOTCH MEDICAL BOARD PREP BIOCHEMISTRY PEARLS DIGITAL HANDOUT BY DRS. RUBIO AND BATICULON Page 18 of 49
For inquiries visit www.topnotchboardprep.com.ph or email us at topnotchmedicalboardprep@gmail.com
This handout is only valid for the March 2023 PLE batch. This will be rendered obsolete for the next batch since we update our handouts regularly.
TOPNOTCH MEDICAL BOARD PREP BIOCHEMISTRY PEARLS DIGITAL HANDOUT BY DRS. RUBIO AND BATICULON
For inquiries visit www.topnotchboardprep.com.ph or https://www.facebook.com/topnotchmedicalboardprep/
This handout is only valid for the March 2023 PLE batch. This will be rendered obsolete for the next batch since we update our handouts regularly.

Regarding collagen, which of the following statements is i. Glycolysis


CORRECT? ii. Gluconeogenesis
A. Synthesis of procollagen from precursor iii. Krebs cycle
polypeptide chains is an extracellular process
B. Procollagen is converted to collagen by cleaving Which pathway/s generate/s NADH?
16.
12. amino acids from the n-terminus A. i and ii
C. It exists mainly in nonfibrous form in the B. i and iii
basement membrane of kidney glomeruli and C. ii only
lens capsule D. iii only
D. Hydroxyproline and hydroxylysine are E. i, ii, and iii
commonly found in the X position
i. Beta-oxidation
ii. Lipogenesis
iii. Ketogenesis

Which pathway/s is/are active during the post-
17. absorptive state?

A. i and ii
B. i and iii
C. ii only
D. iii only
E. i, ii, and iii

i. Glycolysis
ii. Hexose monophosphate shunt
iii. Beta-oxidation

Which pathway/s generate/s ATP?
18.
A. i and ii
B. i and iii
C. ii only

Which of the following is a sugar alcohol used as an D. iii only
artificial sweetener? E. i, ii, and iii
A. Erythritol
13. i. Lipolysis
B. Galactitol
C. Glucuronic acid ii. Krebs cycle
D. Mannitol iii. Hexose monophosphate shunt
Regarding mineral metabolism, which of the following
statements is INCORRECT? Which pathway/s involve/s oxidation reactions?
19.
A. Acrodermatitis enteropathica is treated with A. i and ii
oral zinc supplementation B. i and iii
B. Aside from hypocalcemia, hypomagnesemia can C. ii only
also result in tetany D. iii only
14. C. Deficiency in molybdenum leads to E. i, ii, and iii
accumulation of sulfite, because it is a cofactor of
sulfite oxidase i. Glycolysis
D. Selenium protects against oxidative injury in ii. Gluconeogenesis
tissues iii. Glycogenolysis
E. Synthesis of melanin, catecholamines, and
collagen require iron In the absence of beta-oxidation, which pathway/s will
20. be impaired?

i. Glycolysis A. i and ii
ii. Gluconeogenesis B. i and iii
iii. Glycogenolysis C. ii only
iv. Glycogenesis D. iii only
E. i, ii, and iii
15. Which pathway/s yield/s glucose?

A. i and ii
B. i and iii
C. ii only
D. ii and iii
E. ii and iv
TOPNOTCH MEDICAL BOARD PREP BIOCHEMISTRY PEARLS DIGITAL HANDOUT BY DRS. RUBIO AND BATICULON Page 19 of 49
For inquiries visit www.topnotchboardprep.com.ph or email us at topnotchmedicalboardprep@gmail.com
This handout is only valid for the March 2023 PLE batch. This will be rendered obsolete for the next batch since we update our handouts regularly.
TOPNOTCH MEDICAL BOARD PREP BIOCHEMISTRY PEARLS DIGITAL HANDOUT BY DRS. RUBIO AND BATICULON
For inquiries visit www.topnotchboardprep.com.ph or https://www.facebook.com/topnotchmedicalboardprep/
This handout is only valid for the March 2023 PLE batch. This will be rendered obsolete for the next batch since we update our handouts regularly.
COMPLEX INHIBITOR
CLINICAL CASES Complex I • Barbiturates
A 35-year-old farmer was brought to the emergency room NADH dehydrogenase • Piericidin A
because of profuse sweating, blurring of vision, salivation, (NADH:CoQ • Amytal
and vomiting. He was bradycardic and diaphoretic. His oxidoreductase) • Rotenone
family said he was applying an insecticide just before he
experienced these symptoms. Which type of inhibition • Malonate
1. Complex II
does this poisoning represent? • Carboxin
Succinate dehydrogenase
A. Competitive • TTFA
B. Noncompetitive Complex III
C. Reversible • Antimycin A
Cytochrome b-c1 complex
D. Uncompetitive • Dimercaprol
(CoQ:c1 oxidoreductase)

• Cyanide
Complex IV • Carbon monoxide
Cytochrome c oxidase • Sodium Azide
• Hydrogen Sulfide

A 34-year-old female undergoing general anesthesia for
the first time suddenly developed high fever, muscle
rigidity, and tachycardia. Which of the following explains
her signs and symptoms?
3. A. Cytochrome oxidase is inhibited
B. Electron transport and phosphorylation are
uncoupled
C. NADH dehydrogenase is inhibited
D. There is inadequate supply of ADP
Refer to figure in Question #2

A 47-year-old alcoholic is admitted in the hospital
because of confusion, nystagmus, and difficulty walking.
Which of the following enzymes is expected to be
http://www.tokresource.org/tok_classes/biobiobio/biomenu/enzymes/index.htm
impaired in this patient?
4.
A. α-ketoglutarate dehydrogenase
B. Isocitrate dehydrogenase
C. Malate dehydrogenase
D. Succinate dehydrogenase


Harper’s Biochemistry

A 54-year-old firefighter was brought to the emergency
room from the scene of a fire because of headache,
dizziness, and confusion. He had pink-red discoloration of
his skin and mucous membranes. Which of the following
explains his signs and symptoms?
2.
A. Cytochrome oxidase is inhibited
B. Electron transport and phosphorylation are
uncoupled
C. NADH dehydrogenase is inhibited
D. There is inadequate supply of ADP

TOPNOTCH MEDICAL BOARD PREP BIOCHEMISTRY PEARLS DIGITAL HANDOUT BY DRS. RUBIO AND BATICULON Page 20 of 49
For inquiries visit www.topnotchboardprep.com.ph or email us at topnotchmedicalboardprep@gmail.com
This handout is only valid for the March 2023 PLE batch. This will be rendered obsolete for the next batch since we update our handouts regularly.
TOPNOTCH MEDICAL BOARD PREP BIOCHEMISTRY PEARLS DIGITAL HANDOUT BY DRS. RUBIO AND BATICULON
For inquiries visit www.topnotchboardprep.com.ph or https://www.facebook.com/topnotchmedicalboardprep/
This handout is only valid for the March 2023 PLE batch. This will be rendered obsolete for the next batch since we update our handouts regularly.


A 35-year-old female was brought to the emergency
department because of sudden onset right-sided
weakness with vision loss. She has had previous acute
attacks before, in which the neurologic symptoms would
resolve in 24 to 48 hours. Since she started having these
episodes in the last five years, she has had behavioral
changes and memory problems. On examination, she has
expressive aphasia, right hemiplegia, and right
8. homonymous
hemianopsia. Serum lactate was elevated
and MRI confirmed an acute infarct. Her mother, uncle,
and two other siblings also exhibit the same symptoms,
but the children of her maternal uncle do not. What is the
most likely diagnosis?
A. Homocystinuria
A 32-year-old alcoholic was admitted to a substance B. Leber hereditary optic neuropathy
abuse center. She was given a drug to try to discourage her C. MELAS
from drinking alcohol. Which of the following would D. Methylmalonic aciduria
become elevated and lead to nausea and vomiting if she A 6-year-old girl was brought to a pulmonologist because
5. continues to drink alcohol?
of recurrent respiratory tract infection. She was recently
A. Acetaldehyde hospitalized because of severe bilateral pneumonia, after
B. Acetic acid presenting with wheezing, cough, and fever. Since she was
C. Ethanol an infant, she has had steatorrhea and malabsorption. She
D. Methanol also had a history of intestinal obstruction as a neonate.
9. She is noticeably short and small for her age. Her parents

are not consanguineous and her older brother is healthy.
What is the most likely diagnosis?
A. Chronic granulomatous disease
B. Cystic fibrosis
C. Severe combined immunodeficiency
D. X-linked agammaglobulinemia


Lippincott’s Biochemistry

After breaking up with his girlfriend of 10 years, a 26-


year-old male drank an unspecified number of bottles of
beer. A few hours later, he was brought to the emergency
room unresponsive. Which of the following is LEAST
6. LIKELY to be observed?

A. Hypoglycemia
B. Lactic acidosis
C. Increased acetoacetate
D. Increased NAD+/NADH ratio
A 12-year-old boy was brought to an ophthalmologist by
his mother because of blurring of vision that started 2
months ago. He initially noticed that the vision in his left
eye was blurry, followed by his right eye several weeks
later. On eye examination, he has nystagmus and
7. papilledema.
The most likely diagnosis follows which Pascale Fanen et al. 2014

pattern of inheritance?
A. Autosomal dominant
B. Autosomal recessive
C. X-linked recessive
D. Mitochondrial

TOPNOTCH MEDICAL BOARD PREP BIOCHEMISTRY PEARLS DIGITAL HANDOUT BY DRS. RUBIO AND BATICULON Page 21 of 49
For inquiries visit www.topnotchboardprep.com.ph or email us at topnotchmedicalboardprep@gmail.com
This handout is only valid for the March 2023 PLE batch. This will be rendered obsolete for the next batch since we update our handouts regularly.
TOPNOTCH MEDICAL BOARD PREP BIOCHEMISTRY PEARLS DIGITAL HANDOUT BY DRS. RUBIO AND BATICULON
For inquiries visit www.topnotchboardprep.com.ph or https://www.facebook.com/topnotchmedicalboardprep/
This handout is only valid for the March 2023 PLE batch. This will be rendered obsolete for the next batch since we update our handouts regularly.
A 2-year-old boy was brought to his pediatrician because A 10-month-old infant was taken to his pediatrician
his parents that he would tire easily when playing, often because of failure to achieve developmental milestones.
with shortness of breath. On examination, he is His weight is low for age. He has poor head control, he
tachycardic and pale. He has mild icterus and cannot sit with support or roll over, and his muscle tone
splenomegaly. CBC showed normocytic anemia, with a is generally hypotonic. Chest x-ray showed cardiomegaly.
12.
hematocrit of 24%. Peripheral smear showed spiculated Which enzyme/s is/are most likely to be deficient?
10.
RBCs with elevated reticulocytes. What is the most likely A. 4-4 transferase and/or 1:6 glucosidase
diagnosis? B. Acid α-glucosidase
A. Pyruvate kinase deficiency C. Glucose-6-phosphatase
B. Pyruvate dehydrogenase deficiency D. Skeletal muscle glycogen phosphorylase
C. G6PD deficiency
D. MCAD deficiency



• Glycogen Phosphorylase
• removes glucose-1-P molecules
• shortens the glycogen chain

A 3-month-old infant was taken to her pediatrician
because of worsening lethargy. She was born full term
with no perinatal complications, but was always noted to
be floppy. On examination, she had poor head control and
weak suck. She could not track objects. Laboratory exams
11. showed elevated anion gap, elevated lactate, and elevated

pyruvate. What is the most likely diagnosis?
A. Pyruvate kinase deficiency

B. Pyruvate dehydrogenase deficiency • a(1à4)àa(1à4) glucotransferase
C. G6PD deficiency
• transfers the limit dextrin
D. MCAD deficiency
FATES OF PYRUVATE


• a(1à6) Glucosidase
• removes a FREE glucose by breaking the a(1à6) bond


TOPNOTCH MEDICAL BOARD PREP BIOCHEMISTRY PEARLS DIGITAL HANDOUT BY DRS. RUBIO AND BATICULON Page 22 of 49
For inquiries visit www.topnotchboardprep.com.ph or email us at topnotchmedicalboardprep@gmail.com
This handout is only valid for the March 2023 PLE batch. This will be rendered obsolete for the next batch since we update our handouts regularly.
TOPNOTCH MEDICAL BOARD PREP BIOCHEMISTRY PEARLS DIGITAL HANDOUT BY DRS. RUBIO AND BATICULON
For inquiries visit www.topnotchboardprep.com.ph or https://www.facebook.com/topnotchmedicalboardprep/
This handout is only valid for the March 2023 PLE batch. This will be rendered obsolete for the next batch since we update our handouts regularly.
NAME DEFICIENCY CLINICAL FEATURES
Von Gierke • Glucose-6- • ↑ Glycogen in liver and renal tubule cells, hypoglycemia, lactic acidosis, ketosis,
Ia
disease phosphatase hyperlipemia
• ↑ Glycogen in lysosomes
• Lysosomal acid
II Pompe disease • Juvenile onset: hypotonia, death from heart failure by age 2
maltase
• Adult onset: muscle dystrophy
• Debranching
IIIa Cori disease • Fasting hypoglycemia, hepatomegaly in infancy, ↑ limit dextrin, muscle weakness
enzyme
Andersen • Hepatosplenomegaly,↑ polysaccharide with few branch points, death from heart or
IV • Branching enzyme
disease liver failure before age 5
McArdle • Muscle • Poor exercise tolerance, muscle cramps and myoglobinuria but no lactic acidosis,
V
syndrome phosphorylase á muscle glycogen
• Hepatomegaly,
VI Hers disease • Liver phosphorylase
• mild hypoglycemia

A 9-month-old infant was taken to his pediatrician for A medical student supposed to go on an elective in a
checkup. His parents noticed that his arms and legs don’t malaria-endemic province was prescribed primaquine.
seem to be growing as fast as his face and abdomen. One Shortly after intake of the drug, the student developed
examination, he has a protuberant abdomen from an pallor and anemia. Which of the following is expected in
enlarged liver. He also has plaques on his lower this patient?
15.
extremities and buttocks. Blood glucose level is 28 mg/dL, A. Glucose 6-phosphate dehydrogenase activity is
13. high
with elevated lactate, triglycerides, and uric acid. CBC is
normal. What is the most likely diagnosis? B. Glucose 6-phosphate is low
A. Cori disease C. NADPH is high
B. McArdle disease D. Reduced glutathione is low
C. Pompe disease
D. Von Gierke disease


An 18-year-old female visits a gastroenterologist because
she always experiences diarrhea after drinking coffee
with milk in the morning. She also gets cramps and
flatulence. The most likely reason for this is deficiency of:
16.
A. Salivary amylase
B. Pancreatic amylase
C. Small intestinal lactase
D. Small intestinal isomaltase-sucrase

A 1-year-old boy was brought to his pediatrician because
of vomiting and tremors. Symptoms began after he was
A 16-year-old male complains of recurrent muscle cramps weaned from breast milk. Since then, he has been drinking
and weakness of his legs during basketball. He noticed powdered milk, Yakult, as well as instant orange and
that as he plays longer, the symptoms get worse. What is pineapple juice. Physical examination showed
the most likely diagnosis? hepatomegaly. Laboratory tests showed hypoglycemia
14.
A. Andersen disease 17. and hyperuricemia. His urine was negative for glucose but

B. Hers disease tested positive for reducing sugars. Which enzyme is most
C. McArdle disease likely to be deficient?
D. Von Gierke disease A. Aldolase B
Refer to the table above B. Fructokinase
C. Galactose-1-phosphate uridyltransferase
D. β-galactosidase

TOPNOTCH MEDICAL BOARD PREP BIOCHEMISTRY PEARLS DIGITAL HANDOUT BY DRS. RUBIO AND BATICULON Page 23 of 49
For inquiries visit www.topnotchboardprep.com.ph or email us at topnotchmedicalboardprep@gmail.com
This handout is only valid for the March 2023 PLE batch. This will be rendered obsolete for the next batch since we update our handouts regularly.
TOPNOTCH MEDICAL BOARD PREP BIOCHEMISTRY PEARLS DIGITAL HANDOUT BY DRS. RUBIO AND BATICULON
For inquiries visit www.topnotchboardprep.com.ph or https://www.facebook.com/topnotchmedicalboardprep/
This handout is only valid for the March 2023 PLE batch. This will be rendered obsolete for the next batch since we update our handouts regularly.
An 8-month-old infant was taken to the emergency
department because she was difficult to awaken after 8
hours of sleeping straight. Over the past few weeks, her
parents already noticed that she was becoming more
lethargic. In the ER, she had twitching of her arms and
legs. Physical exam showed hepatomegaly. She had a
18. blood glucose of 24 mg/dL, metabolic acidosis with high

anion gap, and low ketone bodies. What is the most likely
diagnosis?
A. Pyruvate kinase deficiency
B. Pyruvate dehydrogenase deficiency
C. G6PD deficiency
D. MCAD deficiency

7x
OXIDATION
FADH2

THIOLYSIS

HYDRATION

NADH
OXIDATION

7x
CONDENSATION

REDUCTION
NADPH
REDUCTION
NADPH

A newborn infant was diagnosed with congenital adrenal
hyperplasia, resulting from decreased activity of 11β-
hydroxylase. Which of the following is LEAST LIKELY to
be present in this patient?
DEHYDRATION 20.
A. Androstenedione will be high
B. Cortisol will be low
C. Deoxycorticosterone will be high
D. Renin will be high
Refer to the figure for Question #19

A full-term newborn infant was noted to have fused labia A newborn was noted to have multiple congenital
majora and an enlarged clitoris. On the 3rd day of life, the anomalies. She has an extra finger on each hand, a cleft lip
infant was noted to have failure to thrive. She weighed and palate, and Hirschsprung disease. She also has low-set
300 grams less than her ideal weight. Serum sodium and ears, drooping eyelids, and a small upturned nose. What is
potassium were low. Serum 17 α-hydroxyprogesterone 21. the most likely diagnosis?

19. was elevated but 11-deoxycortisol was low. What is the A. 11 β-hydroxylase deficiency
most likely diagnosis? B. 21 α-hydroxylase deficiency
A. 11 β-hydroxylase deficiency C. 7 dehydrocholesterol reductase deficiency
B. 21 α-hydroxylase deficiency D. Phytanoyl CoA hydroxylase deficiency
C. 7 dehydrocholesterol reductase deficiency
D. Phytanoyl CoA hydroxylase deficiency

TOPNOTCH MEDICAL BOARD PREP BIOCHEMISTRY PEARLS DIGITAL HANDOUT BY DRS. RUBIO AND BATICULON Page 24 of 49
For inquiries visit www.topnotchboardprep.com.ph or email us at topnotchmedicalboardprep@gmail.com
This handout is only valid for the March 2023 PLE batch. This will be rendered obsolete for the next batch since we update our handouts regularly.
TOPNOTCH MEDICAL BOARD PREP BIOCHEMISTRY PEARLS DIGITAL HANDOUT BY DRS. RUBIO AND BATICULON
For inquiries visit www.topnotchboardprep.com.ph or https://www.facebook.com/topnotchmedicalboardprep/
This handout is only valid for the March 2023 PLE batch. This will be rendered obsolete for the next batch since we update our handouts regularly.
A 25-year-old call center employee was brought to the
emergency room after experiencing chest heaviness while
talking to a client. The pain radiated to the left shoulder
and left arm. Previously, he would experience the same
symptoms during exertion, but never this severe. He does
not smoke. His maternal grandfather and a maternal uncle
died of a heart attack below 40 years old. ECG showed ST
24. segment
elevation. Troponin I and CK MB were both
elevated. Glucose and uric acid were normal but
triglycerides and LDL were high. What is the most likely
diagnosis?
A. Familial abetalipoproteinemia
B. Familial hyperalphalipoproteinemia
C. Familial hypercholesterolemia
D. Familial lipoprotein lipase deficiency


An 8-year-old was brought to an ophthalmologist because
of blurring of vision, especially at night. Examination
findings were consistent with retinitis pigmentosa. On
further probing, the child also had a poor sense of smell.
Ataxia and dry scaly skin were also noted, prompting
22. referral to a metabolic disease specialist. What enzyme is

most likely to be deficient in this patient?
A. 11 β-hydroxylase
B. 21 α-hydroxylase
C. 7 dehydrocholesterol reductase
D. Phytanoyl CoA hydroxylase
A 5-year-old boy was brought to his pediatrician because HYPERLIPROPROTEINEMIAS
of regression of developmental milestones. He had rapid CLINICAL
neurologic decline, eventually slipping into a coma. Upon DISEASE DEFECT
PRESENTATION
his death, autopsy showed large amounts of very long • High TAG (from
chain fatty acids in his myelin sheath. His adrenal glands elevated chylomicrons),
23. were also abnormal. The impaired enzymatic reaction in
Type I
Deficiency in low LDL, low HDL
this patient occurs in which of the following? Familial
lipoprotein • Xanthomas, recurrent
A. Golgi apparatus Lipoprotein
lipase or pancreatitis,
B. Lysosomes Lipase
apo C-II hepatosplenomegaly
C. Mitochondria Deficiency
• No increased risk of
D. Peroxisomes coronary disease
• High LDL and high
Type IIa Defective
cholesterol
Familial Hyper- LDL
• Atherosclerosis and
cholesterolemia receptor
coronary disease
Type III
• Hypercholesterolemia
Familial Abnormal
• Xanthomas
dysbeta- apo E
• Atherosclerosis
lipoproteinemia
• High VLDL and high TAG
• Subnormal LDL and HDL
Type IV
Over- • Associated with
Familial
production coronary disease, type 2
Hypertriacyl-
of VLDL DM (glucose intolerance,
glycerolemia
hyperinsulinemia),
obesity, alcoholism

A 7-year-old child was brought to the emergency
department because of severe abdominal pain radiating
to the back. She has had two similar episodes in the last
year, but this is when symptoms were severe enough to
prompt consult. On examination, she has small, yellow
deposits of fat under the skin of her trunk, buttocks, and
arms. Abdominal ultrasound revealed
25.
hepatosplenomegaly. Serum triglycerides were elevated
but LDL was low normal. What is the most likely
diagnosis?
A. Familial abetalipoproteinemia
B. Familial hyperalpha-lipoproteinemia
C. Familial hypercholesterolemia
D. Familial lipoprotein lipase deficiency
TOPNOTCH MEDICAL BOARD PREP BIOCHEMISTRY PEARLS DIGITAL HANDOUT BY DRS. RUBIO AND BATICULON Page 25 of 49
For inquiries visit www.topnotchboardprep.com.ph or email us at topnotchmedicalboardprep@gmail.com
This handout is only valid for the March 2023 PLE batch. This will be rendered obsolete for the next batch since we update our handouts regularly.
TOPNOTCH MEDICAL BOARD PREP BIOCHEMISTRY PEARLS DIGITAL HANDOUT BY DRS. RUBIO AND BATICULON
For inquiries visit www.topnotchboardprep.com.ph or https://www.facebook.com/topnotchmedicalboardprep/
This handout is only valid for the March 2023 PLE batch. This will be rendered obsolete for the next batch since we update our handouts regularly.


While applying for a job, an otherwise asymptomatic 32-
year-old male was told that his lipid profile showed an
HDL level that was above the 90th percentile. What should
you consider?
26.
A. Type I hyperlipidemia
B. Type II hyperlipidemia
C. Familial hyperalpha-lipoproteinemia
D. Familial hypercholesterolemia The parents of a 2-week-old male was advised to see a
A 24-year-old female consults a neurologist because she pediatrician because of abnormal newborn screening test
has a family history of blood clots. Her brother died of consistent with phenylketonuria. Which of the following
stroke when he was 18 years old. She is nearsighted and is CORRECT?
she had a hard time in school. On examination, she is tall A. He will exhibit hyperpigmentation
and slender with skinny fingers. Visual acuity is poor and B. His urine will show elevated levels of
30.
her lens is anteriorly displaced. She has mild phenylpyruvate
kyphoscoliosis and pectus excavatum. Rest of C. Restriction of tyrosine should begin
cardiopulmonary findings are unremarkable. CBC, immediately
27.
platelet count, PT/PTT, factor V Leiden, protein C, and D. When the child reaches adulthood,
protein S were within normal limits. Methionine is discontinuation of dietary restrictions is
elevated and cysteine is low. What is the most likely recommended
diagnosis?
A. Galactosemia
B. Homocystinuria
C. Methylmalonic aciduria
D. Phenylketonuria





A 4-year-old boy was noted by his parents to have
darkening of the urine when it was left standing. He has a
normal sibling and there are no other problems. On
examination, he has dark patch on his right earlobe.
Which of the following is most likely to be elevated in this
31.
patient?
A. Homogentisate
B. Methylmalonate
C. Phenylpyruvate
A 24-year-old female with a family history of blood clots D. α-Ketoisovalerate
consulted a neurologist for screening examination.
Because her brother died of stroke at age 17, the
neurologist recommended skin biopsy for fibroblast
culture, which showed deficient activity of the enzyme
28. cystathionine β-synthase. Which of the following should

be recommended for this patient?
A. Increase methionine in the diet
B. Reduce cysteine in the diet
C. Start treatment with vitamin B1
D. Start treatment with vitamin B6
A 2-week-old infant was admitted in the neonatal ICU
because of jaundice, hepatomegaly, and sepsis caused by
gram-negative bacilli. His older sister a similar illness, and
later developed cirrhosis and cataracts. What is the most
29. likely diagnosis?

A. Galactosemia
A 4-day-old infant was brought to the emergency
B. Homocystinuria
department because of lethargy and poor suck. On
C. Methylmalonic aciduria
examination, she had sweet smelling urine. Laboratory
D. Phenylketonuria
workup revealed severe metabolic acidosis. Which of the
32. following will be elevated in this patient’s urine sample?

A. Homogentisate
B. Methylmalonate
C. Phenylpyruvate
D. α-Ketoisovalerate

TOPNOTCH MEDICAL BOARD PREP BIOCHEMISTRY PEARLS DIGITAL HANDOUT BY DRS. RUBIO AND BATICULON Page 26 of 49
For inquiries visit www.topnotchboardprep.com.ph or email us at topnotchmedicalboardprep@gmail.com
This handout is only valid for the March 2023 PLE batch. This will be rendered obsolete for the next batch since we update our handouts regularly.
TOPNOTCH MEDICAL BOARD PREP BIOCHEMISTRY PEARLS DIGITAL HANDOUT BY DRS. RUBIO AND BATICULON
For inquiries visit www.topnotchboardprep.com.ph or https://www.facebook.com/topnotchmedicalboardprep/
This handout is only valid for the March 2023 PLE batch. This will be rendered obsolete for the next batch since we update our handouts regularly.
C. Deficiency in vitamin C
D. Deficiency in prolyl hydroxylase


A 2-year-old child was brought to the emergency
department because of vomiting and irritability. He has
had viral symptoms for the past 3 days. He had similar
episodes before, but this is the most violent he has
A 5-month-old infant was referred to an orthopedic
become. He is febrile, tachycardic, with other signs of
surgeon because of limb shortening and bowing of his
dehydration. On examination, he has hyperreflexia, a mild
long bones. He had an unremarkable perinatal course but
37. flapping tremor when his hands are hyperextended, and a

on his 4th month of life, he developed a fracture of the right
positive Babinski sign. Serum ammonia and urinary orotic
humerus. His sclerae are blue. Chest examination is
acid levels are elevated. What is the most likely diagnosis?
unremarkable. His skin is thin and translucent, showing
A. Arginosuccinate lyase deficiency
33. many bruises. His parents deny any form of child abuse,

B. Arginosuccinate synthetase deficiency
and disclose that other relatives had the same symptoms
C. Carbamoyl phosphate synthetase I deficiency
at a young age. What protein is abnormal in this patient?
D. Ornithine transcarbamylase deficiency
A. Type I Collagen

B. Type III Collagen
On the 2nd day of life, an infant born at home was brought
C. Elastin
to the emergency department because of poor suck and
D. Fibrillin
vomiting. He was lethargic, tachypneic, and hypothermic.

Serum ammonia levels were markedly elevated. What is
A 14-year-old female consulted a neurologist because of
38. the most likely diagnosis?

persistent headache. She experiences pain almost daily,
A. Arginosuccinate lyase deficiency
sometimes awakening her at night. She said that she gets
B. Arginosuccinate synthetase deficiency
bruises easily and her gums often bleed when brushing
C. Carbamoyl phosphate synthetase I deficiency
her teeth. On examination, her skin was elastic and the
D. Ornithine transcarbamylase deficiency
34. joints
of her extremities were lax. What should you
A 3-year-old patient has episodic lethargy and vomiting.
consider?
He was brought to the emergency department where he
A. Alport syndrome
subsequently had seizures. Laboratory exams showed
B. Ehlers Danlos syndrome
elevated serum ammonia and arginosuccinate. What is
C. Epidermolysis bullosa dystrophica
39. the most likely diagnosis?

D. Osteogenesis imperfecta
A. Arginosuccinate lyase deficiency
A 5-month-old infant was brought to a pediatrician for
B. Arginosuccinate synthetase deficiency
poor weight gain. On examination, he has mild icterus, as
C. Carbamoyl phosphate synthetase I deficiency
well as frontal bossing and hepatosplenomegaly.
D. Ornithine transcarbamylase deficiency
Laboratory workup revealed microcytic anemia and a low
reticulocyte count. Hemoglobin electrophoresis revealed
35. markedly decreased HbA. HbF was approximately 70% of

total hemoglobin. What is the most likely diagnosis?
A. α-thalassemia
B. β-thalassemia
C. Hereditary spherocytosis
D. Sickle cell disease

CHAIN FRACTION OF TOTAL
FORM
COMPOSITION HEMOGLOBIN
HbA ⍺2β2 90%
HbF ⍺2γ2 <2%
HbA2 ⍺2δ2 2-5%
HbA1C ⍺2β2-glucose 3-9%

A 27-year-old female consulted a pulmonologist because
of worsening dyspnea. She does not smoke. Her siblings
also complain of “mahina ang baga.” There is no family
history of tuberculosis. Chest x-ray findings were
36.
consistent with emphysema involving both lungs. Which
of the following will explain this patient’s symptoms?
A. Deficiency in α-1 antitrypsin
B. Deficiency in elastase
TOPNOTCH MEDICAL BOARD PREP BIOCHEMISTRY PEARLS DIGITAL HANDOUT BY DRS. RUBIO AND BATICULON Page 27 of 49
For inquiries visit www.topnotchboardprep.com.ph or email us at topnotchmedicalboardprep@gmail.com
This handout is only valid for the March 2023 PLE batch. This will be rendered obsolete for the next batch since we update our handouts regularly.
TOPNOTCH MEDICAL BOARD PREP BIOCHEMISTRY PEARLS DIGITAL HANDOUT BY DRS. RUBIO AND BATICULON
For inquiries visit www.topnotchboardprep.com.ph or https://www.facebook.com/topnotchmedicalboardprep/
This handout is only valid for the March 2023 PLE batch. This will be rendered obsolete for the next batch since we update our handouts regularly.
A 54-year-old male presented with painful blisters on the
back of his hands and nape. This began after he started
running outdoors a couple of weeks ago. He denies using
any new soaps, detergents, or medications. There was no
exposure to any known allergen. He had been on
phenytoin for 2 years now for seizure disorder. Skin
41. culture did not show any organism. 24-hour urine showed

elevated uroporphyrin at 1200 mg. What is the most likely
diagnosis?
A. Acute intermittent porphyria
B. Congenital erythropoietic porphyria
C. Porphyria cutanea tarda
D. Variegate porphyria
A 12-year-old boy was brought to a dermatologist
because of skin that blisters easily. His urine also darkens
on standing and he has stained teeth. Laboratory workup


showed elevated uroporphyrin I and coproporphyrin I in
A 32-year-old female experiences episodes of blood. Uroporphyrin I was also seen in the urine. What is
42.
disorientation, hallucinations, and paranoia. She was the most likely diagnosis?
initially referred to psychiatry, who noted that during A. Acute intermittent porphyria
these attacks, she also had abdominal pain. Laboratory B. Congenital erythropoietic porphyria
testing revealed elevated porphobilinogen and δ- C. Porphyria cutanea tarda
40.
aminolevulinate. What is the most likely diagnosis? D. Variegate porphyria
A. Acute intermittent porphyria
B. Congenital erythropoietic porphyria
C. Porphyria cutanea tarda
D. Variegate porphyria


https://www.slideshare.net/fefoamr/porphyria-29796529

A 3-month-old infant was brought to a pediatrician


because of failure to thrive. He has hepatosplenomegaly.
Dermatan sulfate and heparan sulfate are both elevated in
the urine. He does not have a dazzle reflex and
ophthalmologic examination confirmed corneal clouding.
43. Which
of the following enzymes is most likely to be
deficient?
A. β-glucoronidase
B. β-galactosidase
C. α-L-iduronidase
D. Iduronate sulfatase

A 5-year-old boy is often teased in school because of his
coarse facial features and short stature. On consult with a
pediatrician, he was also noted to have corneal clouding
and hearing loss. His parents disclose that he has
recurrent episodes of upper respiratory infections. His IQ An 8-month-old infant was brought to a pediatrician
44.
is normal for age. What is the most likely diagnosis? because of developmental delay. He is less active
A. Hunter syndrome compared to children of his age. On examination, he has
B. Hurler syndrome fair activity and decreased muscle tone. He cannot roll
C. Morquio syndrome over or sit with support. Physical exam of the chest and
D. Sly syndrome abdomen was unremarkable. Because of a high index of
suspicion, he was referred to an ophthalmologist who
45.
documented a cherry red spot on fundoscopy. Which of
the following enzymes is most likely to be deficient in this
patient?
A. Hexosaminidase A
B. Sphingomyelinase
C. Β-galactosidase
D. β-glucosidase

TOPNOTCH MEDICAL BOARD PREP BIOCHEMISTRY PEARLS DIGITAL HANDOUT BY DRS. RUBIO AND BATICULON Page 28 of 49
For inquiries visit www.topnotchboardprep.com.ph or email us at topnotchmedicalboardprep@gmail.com
This handout is only valid for the March 2023 PLE batch. This will be rendered obsolete for the next batch since we update our handouts regularly.
TOPNOTCH MEDICAL BOARD PREP BIOCHEMISTRY PEARLS DIGITAL HANDOUT BY DRS. RUBIO AND BATICULON
For inquiries visit www.topnotchboardprep.com.ph or https://www.facebook.com/topnotchmedicalboardprep/
This handout is only valid for the March 2023 PLE batch. This will be rendered obsolete for the next batch since we update our handouts regularly.
A 3-year-old child was brought to his pediatrician because
of persistent biting of his fingers. He also likes to bang his
head on the wall for no reason. He is wheelchair-borne,
and he has developmental delay. Laboratory testing
confirmed hyperuricemia. Which of the following
48.
enzymes is most likely deficient?
A. Adenosine deaminase
B. Hypoxanthine guanosine phosphoribosyl transferase
C. Orotidine phosphate decarboxylase
D. Ribonucleotide reductase


SPHINGOLIPIDOSES:
TAY-SACHS DISEASE VS. NIEMANN-PICK DISEASE

A 12-year-old Caucasian was brought to a pediatrician A 51-year-old male undergoing chemotherapy for cancer
because of leg pain especially at night associated with develops severe pain in the metatarsophalangeal joint of
fatigue. On examination, he was noted to have an enlarged his right big toe. On aspiration of his joint fluid,
liver and spleen. Ophthalmologic exam was monosodium urate crystals are seen by polarized light
unremarkable. Which of the following enzymes is most microscopy. His symptoms are due to overproduction of
46. 49.
likely to be deficient in this patient? the end product of which of the following pathways?
A. Hexosaminidase A A. De novo purine synthesis
B. Sphingomyelinase B. De novo pyrimidine synthesis
C. Β-galactosidase C. Purine salvage pathway
D. β-glucosidase D. Purine degradation pathway


SPHINGOLIPIDOSES: GAUCHER DISEASE



A 9-month-infant was brought to a pediatrician because
of high fever and colds that started 3 days ago. Her parents A 2-year-old child is noted to be weak and anemic. Her
said that she has had six or seven such episodes now, but height and weight are low for age and urine showed
this time she appears much sicker. She also had diarrhea elevated levels of orotic acid. Which of the following
a couple of weeks ago. On examination, she is febrile and pathways is impaired in this patient?
50.
tachypneic. She has fallen off the growth curve from the A. De novo purine synthesis
70th percentile in her 6-month visit down to the 20th B. De novo pyrimidine synthesis
percentile for age. Crackles and harsh breath sounds are C. Purine salvage pathway
47. D. Purine degradation pathway
most prominent in the left lung field. CBC showed normal
WBC count and low lymphocyte count. Chest x-ray NUCLEOTIDE SYNTHESIS
showed consolidation in the left lower lobe. Which of the PYRIMIDINE
following enzymes is most likely deficient? PURINE SYNTHESIS
SYNTHESIS
A. Adenosine deaminase Rate- • Carbamoyl
B. Hypoxanthine guanosine phosphoribosyl transferase • PRPP glutamyl
limiting phosphate
C. Orotidine phosphate decarboxylase amidotransferase
enzyme synthetase II
D. Ribonucleotide reductase • Glutamine
• Aspartate
• Glutamine
Donors of • Carbon dioxide
• Aspartate
atoms • Glycine
• Carbon dioxide
• N10-formyltetra-
hydrofolate
Donor of
ribose 5- • PRPP • PRPP
phosphate
• Pyrimidine ring is
• Preformed ribose-
constructed first,
Sequence 5-phosphate comes
then it is attached
of steps first, then purine
to ribose 5-
ring is constructed
phosphate
Parent • Inosine • Orotidine
molecule monophosphate monophosphate
TOPNOTCH MEDICAL BOARD PREP BIOCHEMISTRY PEARLS DIGITAL HANDOUT BY DRS. RUBIO AND BATICULON Page 29 of 49
For inquiries visit www.topnotchboardprep.com.ph or email us at topnotchmedicalboardprep@gmail.com
This handout is only valid for the March 2023 PLE batch. This will be rendered obsolete for the next batch since we update our handouts regularly.
TOPNOTCH MEDICAL BOARD PREP BIOCHEMISTRY PEARLS DIGITAL HANDOUT BY DRS. RUBIO AND BATICULON
For inquiries visit www.topnotchboardprep.com.ph or https://www.facebook.com/topnotchmedicalboardprep/
This handout is only valid for the March 2023 PLE batch. This will be rendered obsolete for the next batch since we update our handouts regularly.

PYRIMIDINE A 35-year-old male was noted to have right-sided


PURINE SYNTHESIS
SYNTHESIS mucinous colon cancer on screening colonoscopy. There
• UMP → UTP → CTP were no other polyps seen. His father and paternal uncle
• UMP → dUMP were also previously diagnosed with colon cancer before
• IMP → GMP age 40. Which of the following is most likely to be
• Ribonucleotide 52.
(Requires ATP) defective in this patient?
Products reductase
• IMP → AMP A. Removal of pyrimidine dimers from DNA
• dUMP → TMP
(Requires GTP) B. Removal of uracil from DNA
• Thymidylate
synthase C. Repair of double strand breaks
Where It • Occurs in the liver, and to a limited extent, in D. Repair of mismatched strands
Occurs the brain


While on picnic, a

family decided to
An 8-year-old girl was brought to a dermatologist because include some wild
she is excessively sensitive to sunlight. She has many mushrooms in their
freckles on sun-exposed parts of her body and she has two salad, after which, all of
lesions on her face, which showed basal cell carcinoma on them became ill. Which
biopsy. Which of the following is most likely to be 53. of the following
51. enzymes was most
defective in this patient?
likely inhibited?
A. Removal of pyrimidine dimers from DNA
B. Removal of uracil from DNA A. DNA gyrase
C. Repair of double strand breaks B. DNA ligase
D. Repair of mismatched strands C. DNA polymerase III
D. RNA polymerase II

A bacterial mutant grows normally at 32OC but at 42OC
accumulates short segments of newly synthesized DNA.
Which of the following enzymes is most likely to be
defective at the nonpermissive temperature?
54.
A. DNA ligase
B. DNA polymerase III
C. DNA primase
D. Topoisomerase

TOPNOTCH MEDICAL BOARD PREP BIOCHEMISTRY PEARLS DIGITAL HANDOUT BY DRS. RUBIO AND BATICULON Page 30 of 49
For inquiries visit www.topnotchboardprep.com.ph or email us at topnotchmedicalboardprep@gmail.com
This handout is only valid for the March 2023 PLE batch. This will be rendered obsolete for the next batch since we update our handouts regularly.
TOPNOTCH MEDICAL BOARD PREP BIOCHEMISTRY PEARLS DIGITAL HANDOUT BY DRS. RUBIO AND BATICULON
For inquiries visit www.topnotchboardprep.com.ph or https://www.facebook.com/topnotchmedicalboardprep/
This handout is only valid for the March 2023 PLE batch. This will be rendered obsolete for the next batch since we update our handouts regularly.


A young male patient was brought to the emergency department because of severe muscle pains throughout his body, which
started after he exercised vigorously. He had two similar episodes years before. CBC showed anemia and peripheral smear
revealed odd-looking RBCs that resembled an elongated sausage rather than concave disks. Which of the following mutations
will most likely explain his findings?
55.
A. Deletion
B. Insertion
C. Missense
D. Nonsense


A 24-year-old patient with multiple sexual partners presents with purple plaques and nodules in the arms and legs. The patient
also has oral thrush and chest findings compatible with pneumonia. If the screening test were positive, which of the following
would confirm your diagnosis?
56. A. Northern blot
B. Polymerase chain reaction
C. Southern blot
D. Western blot

SAMPLE GEL
TECHNIQUE PURPOSE
ANALYZED USED
Southern blot DNA Yes Detects DNA changes
Northern blot RNA Yes Measures mRNA amounts and sizes
Western blot Protein Yes Measures protein amounts
ASO DNA No Detects DNA mutations
cDNA or
Microarray No Measures many mRNA levels at once; detects genomic changes
genomic DNA
Proteins or
ELISA No Detects proteins (antigens) or antibodies, detects genomic changes
antibodies
Measures abundance, distribution, posttranslational modifications, functions, and
Proteomics Proteins Yes
interactions of cellular proteins

A 2-week-old infant diagnosed with type 2 Genetic analysis of a child with β-thalassemia showed that
hyperammonemia had enzyme activity analyzed. There one his β-globin genes has a mutation, in which a new
was no activity for the OTC enzyme. The mRNA product of splice acceptor site is created, 19 nucleotides upstream of
the gene for OTC was identical to that of a control. Which the normal splice acceptor site of the first intron. What
of the following tests was most likely used to analyze the will happen to the new tRNA molecule because of this
57. 58.
size and amount of the mRNA? mutation?
A. Northern blot A. Exon 1 will be shorter
B. Polymerase chain reaction B. Exon 1 will be longer
C. Southern blot C. Exon 2 will be shorter
D. Western blot D. Exon 2 will be longer

TOPNOTCH MEDICAL BOARD PREP BIOCHEMISTRY PEARLS DIGITAL HANDOUT BY DRS. RUBIO AND BATICULON Page 31 of 49
For inquiries visit www.topnotchboardprep.com.ph or email us at topnotchmedicalboardprep@gmail.com
This handout is only valid for the March 2023 PLE batch. This will be rendered obsolete for the next batch since we update our handouts regularly.
TOPNOTCH MEDICAL BOARD PREP BIOCHEMISTRY PEARLS DIGITAL HANDOUT BY DRS. RUBIO AND BATICULON
For inquiries visit www.topnotchboardprep.com.ph or https://www.facebook.com/topnotchmedicalboardprep/
This handout is only valid for the March 2023 PLE batch. This will be rendered obsolete for the next batch since we update our handouts regularly.
EXON 1 INTRON 1 EXON 2 INTRON 2 EXON 3 A blood sample is taken from a patient after an overnight
fast. Which of the following will be at a higher
EXON 1 INTRON 1 EXON 2 INTRON 2 EXON 3 concentration than if the blood were taken after a meal?
62. A. Glucose

B. Insulin
Genetic analysis of a child with Duchenne muscular
C. Nonesterified fatty acids
dystrophy showed a mutation in the promoter region of
D. Triacylglycerol
the DMD gene. What is the most likely effect of this
A blood sample is taken from a patient after fasting for one
mutation?
week, drinking only water. Which of the following will be
A. Capping of dystrophin mRNA will be defective
59. at a higher concentration than if the blood were taken
B. Initiation of dystrophin transcription will be
after an overnight fast?
defective 63.
A. Glucose
C. Splicing of dystrophin mRNA will be defective
B. Ketone bodies
D. Termination of dystrophin transcription will be
C. Nonesterified fatty acids
defective
D. Triacylglycerol
A patient was noted to have an abnormal protein that is
A blood sample was taken from a patient 1 to 2 hours after
180 amino acids instead of 126. Which of the following
eating chicharon, crispy pata, and lechon kawali. Which of
mutations is consistent with this?
the following will be elevated?
60. A. AUG à UAG
64. A. Chylomicrons
B. GAA à UAA
B. HDL
C. UAA à GAA
C. Ketone bodies
D. UAA à UGA
D. VLDL
A blood sample is taken from a patient after eating
A blood sample was taken from a patient 4 to 5 hours after
spaghetti and fried chicken. Which of the following will be
eating chicharon, crispy pata, and lechon kawali. Which of
at a higher concentration than if the blood were taken
the following will be elevated?
after an overnight fast?
61. 65. A. Chylomicrons
A. Alanine
B. HDL
B. Glucagon
C. Ketone bodies
C. Glucose
D. VLDL
D. Ketone bodies

IMPORTANT LIPOPROTEINS
LIPOPROTEIN SOURCE PROTEIN LIPID REMARKS
Largest diameter
Lowest density
Chylomicron Intestine 1–2% 98–99%
Highest
TAG content
VLDL Liver 7–10% 90–93% -
IDL VLDL 11% 89% -
LDL VLDL 21% 79% Highest cholesterol content
Liver
HDL 32–57% 43–68% Highest protein content
Intestine
Note: Free fatty acids or nonesterified fatty acids are transported in the plasma bound to albumin, with low levels during the well-fed state, but which subsequently
rise in the fasted state.


For #64
TOPNOTCH MEDICAL BOARD PREP BIOCHEMISTRY PEARLS DIGITAL HANDOUT BY DRS. RUBIO AND BATICULON Page 32 of 49
For inquiries visit www.topnotchboardprep.com.ph or email us at topnotchmedicalboardprep@gmail.com
This handout is only valid for the March 2023 PLE batch. This will be rendered obsolete for the next batch since we update our handouts regularly.
TOPNOTCH MEDICAL BOARD PREP BIOCHEMISTRY PEARLS DIGITAL HANDOUT BY DRS. RUBIO AND BATICULON
For inquiries visit www.topnotchboardprep.com.ph or https://www.facebook.com/topnotchmedicalboardprep/
This handout is only valid for the March 2023 PLE batch. This will be rendered obsolete for the next batch since we update our handouts regularly.


For #65
Which of the following will have the highest glycemic An alcoholic was found unconscious near a local bar and
index? was taken to the emergency department where
A. Green mango laboratory tests showed a blood glucose level of 40
66.
B. Mango juice mg/dL. To confirm the most likely vitamin deficiency in
C. Mango pie 69. this patient, which diagnostic assay should you order?

D. Boiled camote A. Erythrocyte transaminase assay
Low GI food (55 or less) B. Erythrocyte transketolase assay
• 100% stone-ground whole wheat or pumpernickel bread C. FIGlu excretion test
• Oatmeal (rolled or steel-cut), oat bran, muesli D. Schilling test
• Pasta, converted rice, barley, bulgar Over the past 3 months, a patient excluded all meats, eggs,
• Sweet potato, corn, yam, lima/butter beans, peas, legumes and legumes, nuts, and seeds from her diet. She takes a protein
lentils supplement but not a vitamin supplement. Which of the
• Most fruits, non-starchy vegetables and carrots following will be impaired in this patient?
70.
A. Absorption of amino acids in the intestines
Medium GI (56-69) B. Production of urea by intestinal bacteria
• Whole wheat, rye, and pita bread C. Production of uric acid
• Quick oats D. Transamination of amino acids
• Brown, wild, or basmati rice, couscous A medical student trying to lose weight is doing
intermittent fasting, with his last meal at 6 pm every day.
High GI (70 or more) He does not eat anything until 8 am the next day. The
• White bread or bagel following are observed at 6 am EXCEPT:
71.
• Corn flakes, puffed rice, bran flakes, instant oatmeal A. Increased gluconeogenesis
B. Increased lipolysis
• Shortgrain white rice, rice pasta, macaroni and cheese from mix
C. Increased protein synthesis
• Russet potato, pumpkin
D. Increased glycogenolysis
• Pretzels, rice cakes, popcorn, saltine crackers
A medical student started a ketogenic diet. For lunch, he
• Melons and pineapple
ate eggs, bacon, and greens with olive oil as dressing. The

following are expected after his meal EXCEPT:
Which of the following will have the lowest glycemic A. Gluconeogenesis is activated
index? B. Lipolysis is inhibited
A. Boiled camote 72.
67. C. Acetoacetate and β-hydroxybutyrate will be
B. Camote cue elevated compared with a person on a normal
C. Mango juice diet
D. Mango pie D. Insulin will be decreased compared to a person
Healthy volunteers were fed mixtures of amino acids as on a normal diet
their only protein source. Which of the following diets will A medical student about to take the board exam had hot
lead to negative nitrogen balance, assuming that all other chocolate and pan de sal at 6 am, after which, he was not
amino acids are provided in adequate amounts? able to eat anything else throughout the day. Which of the
68.
A. Mixture lacking alanine, aspartate, glutamate following pathways is expected to primarily maintain his
B. Mixture lacking alanine, glycine, tyrosine 73. blood glucose level at 4 pm?

C. Mixture lacking asparagine, glutamine, cysteine A. Glycogenolysis
D. Mixture lacking lysine, arginine, tyrosine B. Gluconeogenesis
C. Beta-oxidation
D. Ketogenesis



TOPNOTCH MEDICAL BOARD PREP BIOCHEMISTRY PEARLS DIGITAL HANDOUT BY DRS. RUBIO AND BATICULON Page 33 of 49
For inquiries visit www.topnotchboardprep.com.ph or email us at topnotchmedicalboardprep@gmail.com
This handout is only valid for the March 2023 PLE batch. This will be rendered obsolete for the next batch since we update our handouts regularly.
TOPNOTCH MEDICAL BOARD PREP BIOCHEMISTRY PEARLS DIGITAL HANDOUT BY DRS. RUBIO AND BATICULON
For inquiries visit www.topnotchboardprep.com.ph or https://www.facebook.com/topnotchmedicalboardprep/
This handout is only valid for the March 2023 PLE batch. This will be rendered obsolete for the next batch since we update our handouts regularly.
Because of fatigue, the medical student just slept after A 3-year-old boy has episodes of skin rashes that
going home, without eating anything. Which of the spontaneously resolve in a week or so. The parents were
following pathways is expected to primarily maintain his advised by the pediatrician to give him niacin and a high-
blood glucose level at 6 am the next day? protein diet every time this occurred. This shortened the
74.
A. Glycogenolysis symptoms to 1 to 2 days. What is the most likely
77.
B. Gluconeogenesis diagnosis?
C. Beta-oxidation A. Cystinuria
D. Ketogenesis B. Hartnup disease
After taking the board exam, a medical student went C. Menkes disease
straight to Jollibee and ordered C3 (Jolly Spaghetti + D. Wilson disease
Chickenjoy), Large Coke + More Sauce + Extra Rice + Methotrexate toxicity mimics which of the following
Peach Mango Pie. After the meal, the following will be vitamin deficiencies?
75. observed EXCEPT:
A. Biotin
78.
A. Fructose-1,6-bisphosphatase is phosphorylated B. Folate
B. Glycogen synthase is activated C. Pyridoxine
C. HMG CoA synthase is inhibited D. Thiamine
D. Hormone sensitive lipase is dephosphorylated
Elevated homocysteine is associate with deficiency of
which of the following?
A. Ascorbic acid
76.
B. Cobalamin
C. Niacin
D. Tocopherol


Vitamin B12 deficiency leads to trapping of folate in which
form?
A. Dihydrofolate
79.
B. N10-formyl-tetrahydrofolate
C. N5-methyl-tetrahydrofolate
D. N5,N10-methylene-tetrahydrofolate


A patient who ingests 20 raw eggs per day will be expected to have elevated levels of which of the following?
A. Malonate
80. B. Methylmalonate
C. Oxaloacetate
D. Propionate

A 6-month old infant was brought to a pediatrician
because of rashes and hair loss. She was exclusively
breastfed until 3 months and was weaned thereafter. On
examination, the infant had plaques with scaling and
crusting on her perioral region, cheeks, scalp, perineal
area, hands, and feet. She had very sparse hair. She had a
81. low birthweight for her age, and her mother reported that

she frequently had watery stools. The serum level of
which nutrient is most likely to be low in this patient?
A. Copper
B. Iron
C. Manganese
D. Zinc




TOPNOTCH MEDICAL BOARD PREP BIOCHEMISTRY PEARLS DIGITAL HANDOUT BY DRS. RUBIO AND BATICULON Page 34 of 49
For inquiries visit www.topnotchboardprep.com.ph or email us at topnotchmedicalboardprep@gmail.com
This handout is only valid for the March 2023 PLE batch. This will be rendered obsolete for the next batch since we update our handouts regularly.
TOPNOTCH MEDICAL BOARD PREP BIOCHEMISTRY PEARLS DIGITAL HANDOUT BY DRS. RUBIO AND BATICULON
For inquiries visit www.topnotchboardprep.com.ph or https://www.facebook.com/topnotchmedicalboardprep/
This handout is only valid for the March 2023 PLE batch. This will be rendered obsolete for the next batch since we update our handouts regularly.
A 35-year-old female consulted a neurologist because of A neonate was noted to have poor cry and poor activity
flapping tremors of both upper limbs associated with during its first few days of life. Laboratory testing shortly
unsteady gait and slurring of speech. On examination, a after birth showed markedly elevated bilirubin levels at
brown discoloration was noted at the outer margin of 42 mg/dL. What is the most likely diagnosis in this
both corneas. Serum ceruloplasmin was low at 4 mg/dL. 84. patient?

82.
What is the most likely diagnosis? A. Crigler-Najjar syndrome
A. Menkes disease B. Dubin-Johnson syndrome
B. Wilson disease C. Gilbert syndrome
C. Lead poisoning D. Rotor syndrome
D. Hemochromatosis
A 6-year-old child was referred for evaluation of jaundice.
An infant was referred for evaluation of intractable This was not associated with pruritus, abdominal pain, or
seizures. On examination, the baby was also drug intake. Laboratory testing revealed mixed
microcephalic with coarse facial features. Laboratory conjugated and unconjugated hyperbilirubinemia (total
workup showed elevated levels of xanthine and bilirubin: 4 mg/dL) with elevated coproporphyrin in the
hypoxanthine. Which mineral is most likely to be deficient 85. urine. A liver biopsy was not performed. What is the most

83.
in this patient? likely diagnosis in this patient?
A. Copper A. Crigler-Najjar syndrome
B. Manganese B. Dubin-Johnson syndrome
C. Selenium C. Gilbert syndrome
D. Molybdenum D. Rotor syndrome


Gilbert-Meulengracht Dubin-Johnson Rotor
Canalicular multispecific organic
UDP- glucuronosyltransferase
Genetic basis anion transporter ABCC2; Unknown
UGT1A1; chromosome 2q37
chromosome 10q23
Underlying Inappropriate conjugation activity Biliary transport deficiency of non- Defective hepatic storage of
mechanism (uptake? transport?) bile acid organic anions conjugated bilirubin
Inheritance Autosomal recessive Autosomal recessive Autosomal recessive
Hyperbilirubinemia Unconjugated Conjugated = unconjugated Conjugated > unconjugated
Level of Fluctuating, 50-150 uM/l Fluctuating, 50-100 uM/l Fluctuating, 50-100 M/l
hyperbilirubinemia (higher levels possible) (up to >400 possible)
Aminotransferases Normal Normal Normal
Gross pathology: black liver,
Histology Normal Normal
histology: lysosomal pigment
2-5-fold excretion of
Normal urine coproporphyrin Normal total urine coproporphyrin,
Urine features coproporphyrin,
(75% coproporphyrin III) 80% coproporphyrin I
65% coproporphyrin I



END OF BIOCHEMISTRY PEARLS







TOPNOTCH MEDICAL BOARD PREP BIOCHEMISTRY PEARLS DIGITAL HANDOUT BY DRS. RUBIO AND BATICULON Page 35 of 49
For inquiries visit www.topnotchboardprep.com.ph or email us at topnotchmedicalboardprep@gmail.com
This handout is only valid for the March 2023 PLE batch. This will be rendered obsolete for the next batch since we update our handouts regularly.
TOPNOTCH MEDICAL BOARD PREP BIOCHEMISTRY PEARLS DIGITAL HANDOUT BY DRS. RUBIO AND BATICULON
For inquiries visit www.topnotchboardprep.com.ph or https://www.facebook.com/topnotchmedicalboardprep/
This handout is only valid for the March 2023 PLE batch. This will be rendered obsolete for the next batch since we update our handouts regularly.
FLUOROQUINOLONES
FLASHCARDS These antibiotics interfere with (Ciprofloxacin,
IMPORTANT DRUGS AND DISEASES bacterial DNA synthesis by Ofloxacin,
During myocardial ischemia, this inhibiting topoisomerase type II Levofloxacin,
drug inhibits β-oxidation of free Moxifloxacin)
TRIMETAZIDINE
fatty acids, thereby increasing the
metabolic rate of glucose

Inhibits xanthine oxidase, used in


ALLOPURINOL
the treatment of gout

Inhibits HMG CoA reductase, used


in the treatment of STATINS
hypercholesterolemia
Anti-inflammatory drugs that This antineoplastic and immuno-
inhibit phospholipase A2 and suppressant drug inhibits METHOTREXATE
STEROIDS dihydrofolate reductase
decrease the production of
arachidonic acid
NSAIDs This antineoplastic drug inhibits
These drugs decrease inflammation thymidylate synthase and causes 5-FLUOROURACIL
(Mefenamic Acid,
by reversibly inhibiting thymine-less cell death
Ibuprofen,
cyclooxygenase
Ketorolac...)
These drugs decrease inflammation
CELECOXIB,
by selectively inhibiting cyclo-
ETORICOXIB
oxygenase-2

This drug irreversibly inhibits


ASPIRIN
cyclooxygenase

This drug decreases inflammation


ZILEUTON
in asthma by inhibiting lipo-

oxygenase

These drugs are useful in asthma


MONTELUKAST,
because they inhibit leukotriene
ZAFIRLUKAST
receptors

This prostaglandin is useful in Toxicity from this antimyco-
PGE1
impotence and in keeping the bacterial drug may be alleviated by ISONIAZID
ALPROSTADIL
ductus arteriosus patent administration of Vitamin B6
Oral antidiabetic drugs that reduce
postprandial hyperglycemia by Α – GLUCOSIDASE This antimycobacterial drug
inhibiting the conversion of INHIBITORS inhibits DNA-dependent RNA RIFAMPICIN
complex starches, oligosaccharides, (ACARBOSE, polymerase
and disaccharides into mono- MIGLITOL)
saccharides This antibiotic inhibits prokaryotic
These antibiotics interfere with peptidyltransferase and peptide CHLORAMPHENICOL
bacterial DNA synthesis by bond formation
SULFONAMIDES
competitively inhibiting dihydro-
pteroate synthase This antibiotic prevents binding of
TETRACYCLINE
This antibiotic interferes with aminoacyl-tRNAs to the A site
bacterial DNA synthesis by TRIMETHOPRIM
inhibiting dihydrofolate reductase AMINOGLYCOSIDES
These antibiotics bind to the 30S (Streptomycin,
subunit and block the formation of Amikacin,
the initiation complex Gentamicin,
Neomycin...)
CLINDAMYCIN AND
These antibiotics bind to the 50S MACROLIDES
subunit and inhibit translocation (Erythromycin,
Azithromycin...)

Inhibitor of IMP dehydrogenase


MYCOPHENOLATE
used as an immunosuppressant

Vitamin K antagonist WARFARIN

TOPNOTCH MEDICAL BOARD PREP BIOCHEMISTRY PEARLS DIGITAL HANDOUT BY DRS. RUBIO AND BATICULON Page 36 of 49
For inquiries visit www.topnotchboardprep.com.ph or email us at topnotchmedicalboardprep@gmail.com
This handout is only valid for the March 2023 PLE batch. This will be rendered obsolete for the next batch since we update our handouts regularly.
TOPNOTCH MEDICAL BOARD PREP BIOCHEMISTRY PEARLS DIGITAL HANDOUT BY DRS. RUBIO AND BATICULON
For inquiries visit www.topnotchboardprep.com.ph or https://www.facebook.com/topnotchmedicalboardprep/
This handout is only valid for the March 2023 PLE batch. This will be rendered obsolete for the next batch since we update our handouts regularly.

IMPORTANT DISEASES
Galactosemia, galactosuria, GALACTOKINASE
Chronic hemolytic anemia, most cataracts in early childhood DEFICIENCY
PYRUVATE KINASE
common enzyme defect in
DEFICIENCY
glycolysis
FRUCTOKINASE
Essential fructosuria
PYRUVATE DEFICIENCY
Congenital lactic acidosis, X-linked
DEHYDROGENASE
dominant condition
DEFICIENCY
Fructosuria, severe hypoglycemia, FRUCTOSE
lactic acidosis, liver damage, INTOLERANCE
Flatulence, cramps, and diarrhea LACTOSE jaundice Aldolase B Deficiency
after ingestion of dairy products. INTOLERANCE
Progressive cognitive and
behavioral impairment due to
VON GIERKE DISEASE accumulation of amyloid plaques in ALZHEIMER DISEASE
Severe fasting hypoglycemia,
Glucose 6- the hippocampus and cerebral
hepatomegaly, elevated glycogen in
Phosphatase cortex
liver
Deficiency Fatal neurodegenerative diseases
POMPE DISEASE characterized by spongiform
Cardiomegaly and heart failure PRION DISEASES
Lysosomal Acid changes, astrocytic gliomas, and
from impaired glycogen metabolism neuronal loss
Maltase Deficiency
Glutamate is replaced by valine at
CORI DISEASE position 6 of the β-globin chain,
Hepatomegaly, milder form of Von SICKLE CELL DISEASE
Debranching enzyme causing hemoglobin that
Gierke disease polymerizes inside the RBC
deficiency
MCARDLE SYNDROME Synthesis of α-chains is decreased
Skeletal Muscle ALPHA THALASSEMIA
Myoglobinuria with strenuous or absent
Glycogen
exercise
Phosphorylase
Deficiency
Synthesis of β-chains is decreased
GLUOSE-6- BETA THALASSEMIA
Decreased NADPH in RBCs leads to or absent
PHOSPHATE
hemolytic anemia due to poor RBC
DEHYDROGENASE
defense against oxidizing agents
DEFICIENCY Spectrin deficiency causes spherical
HEREDITARY
CHRONIC RBCs that are rapidly culled by the
SPHEROCYTOSIS
Recurrent pyogenic infections due GRANULOMATOUS spleen
to impairment of respiratory burst DISEASE
of neutrophils and monocytes NADPH Oxidase
Blue sclerae, multiple fractures, OSTEOGENESIS
Deficiency
conductive hearing loss IMPERFECTA
CLASSIC
Cataracts within a few days of birth,
GALACTOSEMIA
vomiting and diarrhea after milk
Galactose 1P Berry aneurysms, hyperextensible
ingestion, lethargy, hypotonia, EHLERS-DANLOS
Uridyltransferase skin, hypermobile joints, tendency
mental retardation SYNDROME
Deficiency to bleed

TOPNOTCH MEDICAL BOARD PREP BIOCHEMISTRY PEARLS DIGITAL HANDOUT BY DRS. RUBIO AND BATICULON Page 37 of 49
For inquiries visit www.topnotchboardprep.com.ph or email us at topnotchmedicalboardprep@gmail.com
This handout is only valid for the March 2023 PLE batch. This will be rendered obsolete for the next batch since we update our handouts regularly.
TOPNOTCH MEDICAL BOARD PREP BIOCHEMISTRY PEARLS DIGITAL HANDOUT BY DRS. RUBIO AND BATICULON
For inquiries visit www.topnotchboardprep.com.ph or https://www.facebook.com/topnotchmedicalboardprep/
This handout is only valid for the March 2023 PLE batch. This will be rendered obsolete for the next batch since we update our handouts regularly.

Loose teeth, sore spongy gums, poor


Conjugated hyperbilirubinemia
wound healing, petechiae on skin SCURVY ROTOR SYNDROME
discovered by a Filipino
and mucous membranes

EPIDERMOLYSIS Protein deprivation that is relatively


The skin breaks and blisters as a
BULLOSA greater than the reduction in total KWASHIORKOR
result of minor trauma
DYSTROPHICA calories

Caloric deprivation is relatively


Hereditary nephritis with
ALPORT SYNDROME greater than the reduction in MARASMUS
sensorineural hearing loss
protein

Aortic dilatation, Condition marked by increased


MARFAN SYNDROME CACHEXIA
dolichostenomelia, arachnodactyly protein catabolism

Lipid malabsorption resulting in


Panacinar emphysema and liver Α-1 ANTITRYPSIN increased lipids in feces and
STEATORRHEA
failure DEFICIENCY deficiency of essential fatty acids
and fat-soluble vitamins
Hepatolenticular degeneration from
Alcohol leads to fat accumulation in
accumulation of copper in tissues, WILSON DISEASE FATTY LIVER
the liver
with low levels of ceruloplasmin
Impaired transfer of copper from
intestinal mucosal cells to the blood Cerebrohepatorenal syndrome due ZELLWEGER
MENKES DISEASE
leading to growth retardation, to absence of peroxisomes SYNDROME
mental deficiency, and kinky hair
PHENYLKETONURIA
Musty body odor, mental Defect in peroxisomal activation of
Phenylalanine ADRENO-
retardation, growth retardation, fair VLCFA leads to accumulation of
Hydroxylase LEUKODYSTROPHY
skin, eczema VLCFA in the blood and tissues
Deficiency
MEDIUM-CHAIN ACYL-
ALKAPTONURIA Most common inborn error of beta
Urine turns black upon standing COA
Homogentisate oxidation, sudden infant death
with debilitating arthralgias DEHYDROGENASE
oxidase deficiency syndrome
(MCAD) DEFICIENCY

Decreased pigmentation that


ALBINISM Accumulation of phytanic acid REFSUM DISEASE
increases risk for skin cancer

Hypoglycin from unripe fruit of the


Atherosclerosis, lens subluxation,
akee tree inactivates medium- and JAMAICAN VOMITING
stroke, myocardial infarction, HOMOCYSTINURIA
short-chain acyl CoA dehydro- SICKNESS
osteoporosis, tall stature
genase
Staghorn calculi due to inherited Excess TAGs and chylomicrons in FAMILIAL
defect of renal tubular amino acid CYSTINURIA blood leads to deposition in liver, LIPOPROTEIN LIPASE
transporter skin, pancreas DEFICIENCY

Mental retardation from blocked Elevated LDL cholesterol with


MAPLE SYRUP URINE FAMILIAL HYPER-
degradation of branched-chain increased risk for atherosclerosis
DISEASE CHOLESTEROLEMIA
amino acids and coronary artery disease
Metabolic acidosis, reduced blood METHYLMALONIC
FAMILIAL HYPER-
flow leading to seizure, ACIDEMIA Dyslipoproteinemia that is
ALPHA-
encephalopathy, and stroke in very Methylmalonyl CoA beneficial to health and longevity
LIPOPROTEINEMIA
young patients Mutase Deficiency
Photosensitivity, chronic Accumulation of fat in intestinal
ABETA-
inflammation to overt blistering enterocytes and hepatocytes, with
LIPOPROTEINEMIA
and shearing in exposed areas of PORPHYRIAS deficiency in fat-soluble vitamins

the skin due to defects in heme and essential fatty acids
synthesis
CONGENITAL
ACUTE Failure to thrive, salt-wasting,
ADRENAL
Abdominal pain and neuro- INTERMITTENT hypoglycemia, ambiguous genitalia
HYPERPLASIA
psychiatric symptoms from PORPHYRIA
accumulation of ALA and PBG Uroporphyrinogen I Low plasma cholesterol, elevated
Synthase Deficiency SMITH-LEMLI-OPITZ
7DHC, dysmorphic facial features,
SYNDROME
PORPYHYRIA microcephaly, mental retardation,
Most common porphyria, presents 7-Dehydrocholesterol
CUTANEA TARDA congenital heart disease, other
with photosensitivity and urine that Reductase Deficiency
Uroporphyrinogen malformations, often stillborn
is red to brown in natural light due
Decarboxylase Mental retardation, cherry-red spot
to uroporphyrins
Deficiency on macula, lysosomes with onion
TYPE I CRIGLER- skin, (but no hepatosplenomegaly) TAY-SACHS DISEASE
NAJJAR SYNDROME from accumulation of GM2
Severe congenital jaundice with Complete absence of ganglioside
brain damage hepatic UDP-
glucuronosyl
transferase

TOPNOTCH MEDICAL BOARD PREP BIOCHEMISTRY PEARLS DIGITAL HANDOUT BY DRS. RUBIO AND BATICULON Page 38 of 49
For inquiries visit www.topnotchboardprep.com.ph or email us at topnotchmedicalboardprep@gmail.com
This handout is only valid for the March 2023 PLE batch. This will be rendered obsolete for the next batch since we update our handouts regularly.
TOPNOTCH MEDICAL BOARD PREP BIOCHEMISTRY PEARLS DIGITAL HANDOUT BY DRS. RUBIO AND BATICULON
For inquiries visit www.topnotchboardprep.com.ph or https://www.facebook.com/topnotchmedicalboardprep/
This handout is only valid for the March 2023 PLE batch. This will be rendered obsolete for the next batch since we update our handouts regularly.
Mental retardation, aseptic necrosis Accepts protons, positively charged
of femur, enlarged liver and spleen BASES
GAUCHER DISEASE at physiologic pH, pKa above 7
from accumulation of glucosyl-
ceramide Any substance that resists a change
Mental retardation, enlarged liver in pH when protons are produced or BUFFER
and spleen, cherry red spot on NIEMANN-PICK consumed
macula, foam cells from DISEASE Chemical compound that has a total
ZWITTERION
accumulation of sphingomyelin net charge of zero
Accumulation of dermatan sulfate pH at which the zwitterion is the
and heparin sulfate that leads to predominant form of a chemical ISOELECTRIC pH
mental retardation, coarse facial HUNTER SYNDROME compound.
features, but with NO corneal
This equation is used to calculate HENDERSON-
clouding
the concentration of a weak acid and HASSELBALCH
Accumulation of GAGs that results its conjugate base EQUATION
in skeletal dysplasia, short stature, MORQUIO SYNDROME
but with no CNS involvement IDENTIFY THE BOND
Severe mental retardation, coarse Between a water molecule and
facial features, and skeletal HYDROGEN BOND
another polar compound
abnormalities from accumulation of I-CELL DISEASE
partially degraded glycoproteins in Between monosaccharides in di-,
lysosomes GLYCOSIDIC BOND
oligo-, and polysaccharides

Acute arthritis with deposition of Between amino acids in a protein PEPTIDE BOND
GOUT
uric acid crystals
Between nucleotides in DNA and PHOSPHODIESTER
LESCH-NYHAN RNA BOND
Gout and self-mutilation SYNDROME
HGPRT Deficiency Primary bond of glucosyl residues in α(1,4) GLYCOSIDIC
glycogen BOND
VON GIERKE DISEASE
Glycogen storage disease associated Glucose-6- Branching bond of glucosyl residues α(1,6) GLYCOSIDIC
with gout Phosphatase in glycogen BOND
Deficiency
Between glycerol and its fatty acids ESTER BOND
ADENOSINE
Severe combined
DEAMINASE
immunodeficiency Between two cysteine residues in
DEFICIENCY DISULFIDE BOND
cystine
Between the purine/pyrimidine
Abnormal growth, megaloblastic β-N-GLYCOSIDIC
OROTIC ACIDURIA ring and ribose/deoxyribose in
anemia, orotate in urine BOND
nucleotides
Between C-G and A-T base pairs on
HYDROGEN BOND
DNA
DEFECTS THAT LEAD TO DISEASE

Xeroderma pigmentosum
INABILITY TO REPAIR IMPORTANT BIOMOLECULES
THYMINE DIMERS
Most abundant protein COLLAGEN
INABILITY TO REPAIR
Hereditary nonpolyposis colon Connective tissue protein with
MISMATCHED ELASTIN
cancer rubber-like properties
STRAND

Major fuel of most tissues GLUCOSE


TRIPLE REPEAT
Huntington Disease
EXPANSION
Storage form of glucose GLYCOGEN

CHROMOSOME
Cat cry syndrome Body’s main fuel reserve TRIACYLGLYCEROL
DELETION

Common product of the catabolism


ACETYL COA
of carbohydrates, protein, and lipids
Sickle Cell Disease MISSENSE MUTATION
End-product of mammalian fatty
PALMITATE
acid synthesis
FRAMESHIFT
Duchenne Muscular Dystrophy Precursor of all steroids in the body CHOLESTEROL
MUTATION

The principal high-energy ADENOSINE


compound of the living cell TRIPHOSPHATE
BASIC CHEMISTRY CONCEPTS
INOSINE
Negative logarithm of hydrogen ion Parent purine
pH MONOPHOSPHATE
concentration
Donates protons, negatively OROTIDINE
Parent pyrimidine
charged at physiologic pH, pKa ACIDS MONOPHOSPHATE
below 7
TOPNOTCH MEDICAL BOARD PREP BIOCHEMISTRY PEARLS DIGITAL HANDOUT BY DRS. RUBIO AND BATICULON Page 39 of 49
For inquiries visit www.topnotchboardprep.com.ph or email us at topnotchmedicalboardprep@gmail.com
This handout is only valid for the March 2023 PLE batch. This will be rendered obsolete for the next batch since we update our handouts regularly.
TOPNOTCH MEDICAL BOARD PREP BIOCHEMISTRY PEARLS DIGITAL HANDOUT BY DRS. RUBIO AND BATICULON
For inquiries visit www.topnotchboardprep.com.ph or https://www.facebook.com/topnotchmedicalboardprep/
This handout is only valid for the March 2023 PLE batch. This will be rendered obsolete for the next batch since we update our handouts regularly.

IDENTIFY THE AMINO ACID Predominant free amino acids in


ALANINE, GYCINE
blood
Smallest side chain GLYCINE
CYSTEINE,
Amino acids with sulfur
METHIONINE
Largest side chain TRYPTOPHAN
THREONINE,
Amino acids with hydroxyl groups
TYROSINE, SERINE
Imino acid PROLINE
ISOLEUCINE, LEUCINE,
Branched-chain amino acids
VALINE
Plays a role in oxygen binding to
HISTIDINE TYROSINE,
hemoglobin and myoglobin
PHENYLALANINE,
Aromatic amino acids
TRYPTOPHAN,
Transfer of methyl groups as SAM METHIONINE
HISTIDINE
Used in the diagnosis of folic acid Amino acids with impaired renal CYSTINE, ORNITHINE,
HISTIDINE reabsorption in cystinuria LYSINE, ARGININE
deficiency
VALINE, ISOLEUCINE,
Amino acids whose metabolism is
First step in heme synthesis GLYCINE THREONINE,
impaired in methylmalonic aciduria
METHIONINE

Precursor of niacin TRYPTOPHAN 21st amino acid SELENOCYSTEINE

PVT TIM HALL, always


Precursor of nitric oxide ARGININE Nutritionally essential amino acids
ARGues, never TYRes

Precursor of melanin TYROSINE Purely ketogenic amino acids LEUCINE, LYSINE

TRYPTOPHAN,
Precursor of serotonin TRYPTOPHAN Both glucogenic and ketogenic PHENYLALANINE,
amino acids ISOLEUCINE,
TYROSINE
Precursor of catecholamines TYROSINE
Purely glucogenic amino acids ALL OTHERS
Precursor of melatonin TRYPTOPHAN

PROTEIN STRUCTURE
Precursor of GABA GLUTAMATE
SECONDARY
α-helix and β-pleated sheet
STRUCTURE
Precursor of histamine HISTIDINE
Sequence of amino acids PRIMARY STRUCTURE
Precursor of thyroxine TYROSINE
Determines whether protein is TERTIARY
globular or fibrous STRUCTURE
Precursor of tyrosine PHENYLALANINE
QUATERNARY
More than one polypeptide
STRUCTURE
Precursor of cysteine METHIONINE
Determined using Edman’s reagent
PRIMARY STRUCTURE
and Sanger’s reagent
Synthesized in the urea cycle ARGININE
TERTIARY
Domains
STRUCTURE
Precursor of homocysteine METHIONINE
SUPERSECONDARY
Motifs
STRUCTURE
Responsible for the savory taste
GLUTAMATE
umami

Amino acid in hemoglobin that is LYSINE (ε-AMINO MYOGLOBIN VS HEMOGLOBIN


glycated GROUP)
Contains heme BOTH
Amino acid that is phosphorylated
in enzymes, also the site of O-linked SERINE
glycosylation Fibrous protein NEITHER
Site of N-linked glycosylation of
ASPARAGINE
proteins Only exhibits tertiary structure MYOGLOBIN

Only amino acid that is not chiral GLYCINE


Found in heart and skeletal muscle MYOGLOBIN
GLUTAMATE,
Three amino acids in glutathione Binds four molecules of oxygen at a
CYSTEINE, GLYCINE HEMOGLOBIN
time
ASPARTATE,
Acidic amino acids Binding of oxygen is affected by
GLUTAMATE
changes in pH and CO2 HEMOGLBOIN
HISTIDINE, ARGININE, concentration
Basic amino acids
LYSINE

TOPNOTCH MEDICAL BOARD PREP BIOCHEMISTRY PEARLS DIGITAL HANDOUT BY DRS. RUBIO AND BATICULON Page 40 of 49
For inquiries visit www.topnotchboardprep.com.ph or email us at topnotchmedicalboardprep@gmail.com
This handout is only valid for the March 2023 PLE batch. This will be rendered obsolete for the next batch since we update our handouts regularly.
TOPNOTCH MEDICAL BOARD PREP BIOCHEMISTRY PEARLS DIGITAL HANDOUT BY DRS. RUBIO AND BATICULON
For inquiries visit www.topnotchboardprep.com.ph or https://www.facebook.com/topnotchmedicalboardprep/
This handout is only valid for the March 2023 PLE batch. This will be rendered obsolete for the next batch since we update our handouts regularly.

IDENTIFY THE CARBOHYDRATE


Reservoir of oxygen MYOGLOBIN
Oxidized glucose GLUCURONIC ACID
Exists in taut and relaxed form HEMOGLOBIN
Reduced glucose, responsible for
the complications of diabetes SORBITOL
Sigmoidal oxygen dissociation curve HEMOGLOBIN mellitus
Polysaccharide in plants that cannot
CELLULOSE
be digested by humans
HEMOGLOBIN
Excreted in essential pentosuria XYLULOSE
Most abundant form in adults HbA

Hemoglobin bound to 4 oxygen Component of nucleic acids RIBOSE


R FORM
molecules

Hemoglobin without any oxygen
T FORM IDENTIFY THE RELATIONSHIP BETWEEN THE
molecule bound to it TWO
Non-enzymatic addition of glucose
to hemoglobin, used to determine HBA1C α-D-fructose and β-D-fructose ANOMERS
level of control of diabetes mellitus
Oxidation of the heme component of
hemoglobin to Fe3+, which cannot METHEMOGLOBIN Galactose and Mannose ISOMERS
bind oxygen
ISOMERS AND
Carbon monoxide binds tightly but CARBOXY- Glucose and Mannose
EPIMERS
reversibly to the hemoglobin iron HEMOGLOBIN
Carbon dioxide bound to D-Galactose and L-Galactose ENANTIOMERS
CARBAMINO-
hemoglobin for transport in the
HEMOGLOBIN
blood
Tetramer consisting of two α-chains
and γ-chains
FETAL HEMOGLOBIN IDENTIFY THE GLUT
Requires insulin GLUT-4
γ-tetramers in the newborn HEMOGLOBIN BARTS Brain and RBC GLUT-1
Liver and pancreas GLUT-2
Absorption of fructose in small
Two α and two δ chains HbA2 GLUT-5
intestine by facilitated diffusion
Adipose tissue, skeletal and cardiac
GLUT-4
muscle
COLLAGEN Brain, kidney, placenta GLUT-3

Most common TYPE I
LIPIDS AND LIPOPROTEINS
IDENTIFY THE LIPID
Basement membrane TYPE IV
Long chain of carboxylic acid with SATURATED FATTY
no double bond ACID
Granulation tissue TYPE III
Long chain of carboxylic acid with MONOUNSATURATED
one double bond FATTY ACID
Reticulin, blood vessels TYPE III
Long chain of carboxylic acid with POLYUNSATURATED
two or more double bonds FATTY ACID
Nucleus pulposus, vitreous body,
TYPE II Fatty acids that are byproducts of
and cartilage
hydrogenation, which makes them TRANS-FATTY ACIDS
more solid and “spreadable”
Tendon, fascia, bone TYPE I
TRANS-FATTY ACIDS
Fatty acids associated with
and SATURATED
increased risk of atherosclerosis
Late wound repair TYPE I FATTY ACIDS
Predominant fatty acid in coconut
LAURIC ACID
Beneath stratified squamous oil
TYPE VII
epithelium
LINOLEIC ACID
Essential fatty acids
LINOLENIC ACID
CARBOHYDRATES
Immediate precursor of
CLASSIFY THE FOLLOWING CARBOHYDRATES prostaglandins
ARACHIDONIC ACID

Maltose, sucrose, lactose DISACCHARIDE End product of fatty acid synthesis PALMITIC ACID

Mannose, fructose, galactose MONOSACCHARIDE Major storage form of fatty acids TRIACYLGLYCEROL

3-carbon compound that is a


Cellulose, starch, glycogen, inulin POLYSACCHARIDE product of the oxidation of odd- PROPIONYL CoA
numbered fatty acids
ACETOACETATE
β-
Enumerate the 3 ketone bodies
HYDROXYBUTYRATE
ACETONE
TOPNOTCH MEDICAL BOARD PREP BIOCHEMISTRY PEARLS DIGITAL HANDOUT BY DRS. RUBIO AND BATICULON Page 41 of 49
For inquiries visit www.topnotchboardprep.com.ph or email us at topnotchmedicalboardprep@gmail.com
This handout is only valid for the March 2023 PLE batch. This will be rendered obsolete for the next batch since we update our handouts regularly.
TOPNOTCH MEDICAL BOARD PREP BIOCHEMISTRY PEARLS DIGITAL HANDOUT BY DRS. RUBIO AND BATICULON
For inquiries visit www.topnotchboardprep.com.ph or https://www.facebook.com/topnotchmedicalboardprep/
This handout is only valid for the March 2023 PLE batch. This will be rendered obsolete for the next batch since we update our handouts regularly.

Ketone body detected in urine test IDENTIFY THE LIPASE


ACETOACETATE
for ketones
Degradation of TAG stored in HORMONE SENSITIVE
Ketone body that cannot be used as adipocytes LIPASE
ACETONE
fuel
Degradation of dietary TAG in small
Two molecules from triacylglycerol PANCREATIC LIPASE
GLYCEROL, intestine
degradation that can undergo
PROPIONYL COA Degradation of TAG circulating in
gluconeogenesis LIPOPROTEIN LIPASE
chylomicrons and TAGs
7-DEHYDRO-
Precursor of vitamin D in the skin
CHOLESTEROL Degradation of TAG remaining in
HEPATIC LIPASE
IDL
Precursor of all steroid hormones
PREGNENOLONE
derived from cholesterol

Major source of estrogen in


PURINE VS. PYRIMIDINE
ESTRONE
postmenopausal women
Cytosine, Uracil, Thymine PYRIMIDINES
Means through which cholesterol is
BILE
excreted from the body
Adenine, Guanine, Hypoxanthine PURINES
Principal sterol in the feces COPROSTANOL
Ring is constructed on a preformed
PURINES
CHOLIC ACID ribose 5-phosphate
Give 2 primary bile acids CHENODEOXYCHOLIC
APRT and HGPRT for salvage
ACID PURINES
pathway
Give 2 molecules conjugated to bile
TAURINE, GLYCINE PRPP is donor of ribose 5-
acids to convert them to bile salts BOTH
phosphate
LITHOCHOLIC ACID
Give 2 secondary bile acids Ring can be opened and degraded to
DEOXYCHOLIC ACID PYRIMIDINES
highly soluble structures
Spherical macromolecular
complexes composed of lipids and LIPOPROTEINS
specific proteins Converted to uric acid PURINES

Protein moiety of lipoproteins APOPROTEINS


ENZYMES
IDENTIFY THE LIPOPROTEIN LOWER ENERGY OF
Mechanism of action of enzymes
Transport dietary triglyceride and ACTIVATION
CHYLOMICRON
cholesterol from intestine to tissues This equation describes how
MICHAELIS-MENTEN
VLDL reaction velocity varies with
Transports triglyceride from liver to EQUATION
substrate concentration
tissues This is a double reciprocal plot used

to calculate Km and Vmax, as well as LINEWEAVER-BURK
Delivers cholesterol into cells LDL to determine the mechanism of PLOT
action of enzyme inhibitors
Substrate concentration at which
Reverse cholesterol transport HDL MICHAELIS
the reaction velocity is equal to ½
CONSTANT
Vmax
Shuttles apo C-II and apo E in the
HDL Maximal velocity of an enzyme-
blood Vmax
catalyzed reaction

Highest triglyceride content CHYLOMICRON


ENZYME INHIBITORS
Highest protein content HDL The structure of the inhibitor COMPETITIVE
resembles the substrate INHIBITION

Highest cholesterol content LDL NONCOMPETITIVE


Vmax is lowered
INHIBITION

Largest diameter CHYLOMICRON NONCOMPETITIVE


Km does not change
INHIBITION

Affinity of enzyme to the substrate COMPETITIVE
IDENTIFY THE APOPROTEIN is decreased INHIBITION
Mediates chylomicron secretion Apo B-48 The inhibitor and the substrate bind NONCOMPETITIVE
at different sites on the enzyme INHIBITION
Activates lipoprotein lipase Apo C-II Malathion’s effect on NONCOMPETITIVE
acetylcholinesterase INHIBITION
Mediates uptake of chylomicron
Apo E Simvastatin’s effect on HMG-CoA COMPETITIVE
remnant
reductase INHIBITION
Binds to LDL receptor and mediates
Apo B-100
VLDL secretion

Activates LCAT to produce
Apo A-1
cholesteryl esters in HDL
TOPNOTCH MEDICAL BOARD PREP BIOCHEMISTRY PEARLS DIGITAL HANDOUT BY DRS. RUBIO AND BATICULON Page 42 of 49
For inquiries visit www.topnotchboardprep.com.ph or email us at topnotchmedicalboardprep@gmail.com
This handout is only valid for the March 2023 PLE batch. This will be rendered obsolete for the next batch since we update our handouts regularly.
TOPNOTCH MEDICAL BOARD PREP BIOCHEMISTRY PEARLS DIGITAL HANDOUT BY DRS. RUBIO AND BATICULON
For inquiries visit www.topnotchboardprep.com.ph or https://www.facebook.com/topnotchmedicalboardprep/
This handout is only valid for the March 2023 PLE batch. This will be rendered obsolete for the next batch since we update our handouts regularly.

BIOENENERGETICS HOW DO THE FOLLOWING COMPOUNDS INHIBIT


What is the formula for standard
ATP PRODUCTION?
ΔG = ΔH - TΔS
free energy?
Aspirin UNCOUPLER
Measure of energy available to do
ΔG
work COMPLEX I
Amytal, rotenone, barbiturates
INHIBITOR
Measure of randomness ENTROPY
DIRECT INHIBITION
Oligomycin
OF ATP SYNTHASE
Measure of heat released or
ENTHALPY CYTOCHROME
absorbed during a reaction Carbon monoxide, cyanide,
OXIDASE (COMPLEX
Free energy change under standard hydrogen sulfide
IV) INHIBITOR
conditions ΔGO
(reactants and products at 1 mol/L) COMPLEX III
Dimercaprol, Antimycin A
For a reaction to be spontaneous INHIBITOR
and favorable, what should be the NEGATIVE
COMPLEX II
value of ΔG? Malonate, TTFA
INHIBITOR
Rate of formation of products is the
same as rate of formation of ZERO INHIBITS TRANSPORT
reactants, what is the value of ΔG? Atractyloside
OF ADP AND ATP
OXIDATIVE

Give the two ways in which your PHOSPHORYLATION
cells produce ATP SUBSTRATE LEVEL IDENTIFY THE PATHWAY DESCRIBED
PHOSPHORYLATION Conversion of glucose to 2
Two requirements of oxidative OXYGEN and molecules of either pyruvate or GLYCOLSIS
phosphorylation MITOCHONDRIA lactate
EMBDEN-MEYERHOF-
Two pathways where substrate GLYCOLYSIS Most common type of glycoslysis
PARNAS PATHWAY
level phosphorylation occurs CITRIC ACID CYCLE
Production of glucose from
This theory explains how the free precursors such as lactate, glycerol,
MITCHELL
energy generated by the transport
CHEMIOSMOTIC glucogenic amino acids, GLUCONEOGENESIS
of electrons by the ETC is used to intermediates of glycolysis and TCA
HYPOTHESIS
produce ATP from ADP + Pi.
cycle

Synthesis of storage form of
SUBSTRATE LEVEL OR OXIDATIVE carbohydrates from UDP-glucose
GLYCOGENESIS
PHOSPHORYLATION?
Synthesis of 2,3- RAPOPORT-
SUBSTRATE LEVEL bisphosphoglycerate LUEBERING SHUNT
Anaerobic glycolysis
PHOSPHORYLATION
Retrieval of glucose from its storage
Coupled with the GLYCOGENOLYSIS
OXIDATIVE PHOSPHORYLATION form
electron transport chain
PENTOSE PHOSPHATE
Produces NADPH, ribose 5P, and
Only method of ATP SUBSTRATE LEVEL PATHWAY / HEXOSE
provides a mechanism for metabolic
production for RBCs PHOSPHORYLATION MONOPHOSPHATE
use of 5C sugars
SHUNT
Resting skeletal muscle
and cardiac muscle, Synthesis of acidic sugars, and in URONIC ACID
OXIDATIVE PHOSPHORYLATION some animals, ascorbic acid PATHWAY
well-oxygenated
neurons
Fatigued skeletal Synthesis of sorbitol POLYOL PATHWAY
muscle, cardia muscle
SUBSTRATE LEVEL Final common pathway for the CITRIC ACID CYCLE /
during acute MI, and
PHOSPHORYLATION aerobic oxidation of carbohydrates, TRICARBOXYLIC ACID
neurons during
ischemic stroke lipids, and proteins CYCLE / KREBS CYCLE
Reduced coenzymes NADH and
FADH2 each donate a pair of ELECTRON
ETC AND ATP SYNTHASE electrons to a specialized set of TRANSPORT CHAIN
electron carriers
Entry point for NADH COMPLEX I
Conversion of excess nitrogen into a
non-toxic, highly water-soluble UREA CYCLE
Entry point for FADH2 COMPLEX II compound in humans
KREBS-HENSELEIT
Other name of the urea cycle
CYCLE
Generation of proton gradient COMPLEXES I, III, IV
Synthesis of palmitate from acetyl FATTY ACID
ATP production by oxidative CoA SYNTHESIS
COMPLEX V
phosphorylation
Breakdown of palmitate into acetyl
BETA-OXIDATION
CoA
Non-protein component COENZYME Q
Synthesis of alternative metabolic
KETOGENESIS
COENZYME Q fuel from acetyl CoA during a fast
Mobile components of ETC
CYTOCHROME C Lactate in exercising muscle is
converted back to glucose in the CORI CYCLE
Final electron acceptor OXYGEN liver

TOPNOTCH MEDICAL BOARD PREP BIOCHEMISTRY PEARLS DIGITAL HANDOUT BY DRS. RUBIO AND BATICULON Page 43 of 49
For inquiries visit www.topnotchboardprep.com.ph or email us at topnotchmedicalboardprep@gmail.com
This handout is only valid for the March 2023 PLE batch. This will be rendered obsolete for the next batch since we update our handouts regularly.
TOPNOTCH MEDICAL BOARD PREP BIOCHEMISTRY PEARLS DIGITAL HANDOUT BY DRS. RUBIO AND BATICULON
For inquiries visit www.topnotchboardprep.com.ph or https://www.facebook.com/topnotchmedicalboardprep/
This handout is only valid for the March 2023 PLE batch. This will be rendered obsolete for the next batch since we update our handouts regularly.
In muscle, pyruvate is IDENTIFY THE ORGAN/S WHERE THE GIVEN
transaminated to form alanine,
which is shuttled to the liver where PATHWAYS OCCUR
GLUCOSE – ALANINE
the amino group enters the urea
CYCLE Glycolysis ALL ORGANS
cycle and the pyruvate is used to
make glucose, which is shunted
back to muscle
Glycogen synthesis LIVER and MUSCLE
Provides the glycerol backbone of
triacylglycerol in liver and adipose
GLYCOLYSIS
tissue, and carbon skeleton of Gluconeogenesis LIVER and KIDNEY
nonessential amino acids
Provides the energy required by LIVER and ADIPOSE
BETA OXIDATION Triacylglycerol synthesis
gluconeogenesis TISSUE
ANAEROBIC LIVER and
Only two pathways present in GLYCOLYSIS PENTOSE Cholesterol synthesis
INTESTINES
erythrocytes PHOSPHATE
PATHWAY
Reconvert purines obtained from Ketogenesis LIVER
PURINE SALVAGE
diet and turnover of cellular nucleic
PATHWAY
acids to nucleoside triphosphates Urea Cycle LIVER

RATE-LIMITING ENZYMES ALL CELLS WITH
Electron transport chain MITOCHONDRIA and
PHOSPHO- SUFFICIENT OXYGEN
Glycolysis
FRUCTOKINASE-1 RBCs and TISSUES
Hexose Monophosphate Shunt THAT PRODUCE
FRUCOSE-1,6-
Gluconeogenesis LIPIDS
BISPHOSPHATASE

Glycogenesis GLYCOGEN SYNTHASE IDENTIFY THE SUBCELLULAR LOCATION OF
THE GIVEN PATHWAY
GLYCOGEN
Glycogenolysis Glycolysis CYTOSOL
PHOSPHORYLASE
GLUCOSE-6-
Hexose Monophosphate Shunt PHOSPHATE MITOCHONDRIAL
TCA Cycle
DEHYDROGENASE MATRIX
ISOCITRATE INNER
TCA Cycle Electron transport chain MITOCHONDRIAL
DEHYDROGENASE
MEMBRANE
ACETYL CoA MITOCHONDRIA and
Lipogenesis Gluconeogenesis
CARBOXYLASE CYTOSOL
CARNITINE
Lipolysis PALMITOYL Hexose monophosphate shunt CYTOSOL
TRANSFERASE

Ketogenesis HMG CoA SYNTHASE Fatty acid synthesis CYTOSOL

Cholesterol Synthesis HMG CoA REDUCTASE Beta oxidation MITOCHONDRIA

SMOOTH
Bile Acid Synthesis 7-α HYDROXYLASE Further elongation and
ENDOPLASMIC
desaturation of fatty acids
RETICULUM
CYTOSOL, SMOOTH
Steroid Hormone Synthesis DESMOLASE
Cholesterol synthesis ENDOPLASMIC
CARBAMOYL RETICULUM
Urea Cycle PHOSPHATE Degradation of glycogen by acid
LYSOSOMES
SYNTHETASE I maltase

Heme Synthesis ALA SYNTHASE Oxidation of very long chain fatty


PEROXISOME
acids
PRPP-GLUTAMYL MITOCHONDRIA and
De Novo Purine Synthesis Heme synthesis
AMIDOTRANSFERASE CYTOSOL
CARBAMOYL SMOOTH
De Novo Pyrimidine Synthesis PHOSPHATE Triacylglycerol synthesis ENDOPLASMIC
SYNTHETASE II RETICULUM
TYROSINE
Catecholamine Synthesis Protein synthesis RIBOSOMES
HYDROXYLASE
Degradation of sphingolipids,
glycosaminoglycans, glycolipids, LYSOSOMES
and glycogen






TOPNOTCH MEDICAL BOARD PREP BIOCHEMISTRY PEARLS DIGITAL HANDOUT BY DRS. RUBIO AND BATICULON Page 44 of 49
For inquiries visit www.topnotchboardprep.com.ph or email us at topnotchmedicalboardprep@gmail.com
This handout is only valid for the March 2023 PLE batch. This will be rendered obsolete for the next batch since we update our handouts regularly.
TOPNOTCH MEDICAL BOARD PREP BIOCHEMISTRY PEARLS DIGITAL HANDOUT BY DRS. RUBIO AND BATICULON
For inquiries visit www.topnotchboardprep.com.ph or https://www.facebook.com/topnotchmedicalboardprep/
This handout is only valid for the March 2023 PLE batch. This will be rendered obsolete for the next batch since we update our handouts regularly.

IDENTIFY THE SHUTTLE The liver is able to make ketone


bodies but cannot utilize them due THIOPHORASE
Transport of cytosolic NADH to the GLYCEROL to absence of this enzyme
inner mitochondrial membrane, PHOSPHATE
Muscle unable to provide free
present in brain and white muscle SHUTTLE GLUCOSE 6-
glucose from glycogenolysis due to
Transport of cytosolic NADH to the PHOSPHATASE
lack of this enzyme
inner mitochondrial membrane, MALATE-ASPARTATE
Schwann cells, kidney, and retina
present in heart muscle and most SHUTTLE SORBITOL
lack this enzyme to metabolize
tissues DEHYDROGENASE
sorbitol causing damage
Transport of mitochondrial acetyl-
Enzyme deficient in humans which
CoA into the cytoplasm to produce a CITRATE SHUTTLE L-GULONOLACTONE
is why we cannot synthesize
16-carbon fatty acid OXIDASE
vitamin C
Transport of cytosolic palmitoyl-
CoA into the mitochondria for beta- CARNITINE SHUTTLE Creates α(1-4) linkages and
GLYCOGEN SYNTHASE
oxidation elongates the glycogen chains

Transfers 5 to 8 glucosyl residues
BRANCHING ENZYME
HOW MANY ATPS? and creates α(1-6) linkages
ATPs produced from anaerobic
2 Cleaves α(1-4) bonds to produce GLYCOGEN
glycolysis
glucose 1P PHOSPHORYLASE
ATPs produced from aerobic
5 or 7 (6 or 8)
glycolysis Cleaves α(1-4) and α(1-6) bonds to DEBRANCHING
ATPs produced from each molecule produce free glucose ENZYME
1.5 (2)
of FADH2 in TCA
HEXOKINASE /
ATPs produced from each molecule
2.5 (3) GLUCOKINASE
of NADH in TCA
Irreversible enzymes of glycolysis PHOSPHO-
ATPs produced from complete
30 or 32 (36 or 38) FRUCTOKINASE-1
oxidation of glucose
PYRUVATE KINASE
ATPs produced from complete
106 (129) Major anaplerotic reaction of the PYRUVATE
oxidation of palmitate
ATPs required to create 1 molecule 3 ATPs but 4 high- citric acid cycle CARBOXYLASE
of urea energy bonds PYRUVATE
ATPs required to activate fatty acids 2 ATP equivalents CARBOXYLASE
(i.e., attach CoA) (ATPàAMP) Enzymes of gluconeogenesis that PEP CARBOXYKINASE
ATPs required to convert ribose-5P 2 ATP equivalents reverse the irreversible steps in FRUCTOSE-1,6-
to PRPP (ATPàAMP) glycolysis BISPHOSPHATASE
ATPs required to attach one amino GLUCOSE-6-
acid to the growing polypeptide 4 PHOSPHATASE
chain during translation Through this enzyme, amino acids
transfer their amino groups to AMINOTRANSFERASES
IDENTIFY THE ENZYMES glutamate
Through this enzyme, glutamate is
GLUCOSE-6- GLUTAMATE
Most common disease-producing oxidatively deaminated to liberate
PHOSPHATE DEHYDROGENASE
enzyme deficiency in humans free ammonia
DEHYDROGENASE Through this enzyme, glutamine is
GLUTAMINE
PYRUVATE synthesized from glutamate and
Pyruvate à Acetyl CoA SYNTHETASE
DEHYDROGENASE ammonia
Through this enzyme, glutamine is
LACTATE deaminated to glutamate in the GLUTAMINASE
Pyruvate à Lactate
DEHYDROGENASE kidneys and intestines
PYRUVATE Ribonucleotides à RIBONUCLEOTIDE
Pyruvate à Oxaloacetate
CARBOXYLASE Deoxyribonucleotides REDUCTASE
PYRUVATE ADENOSINE
Pyruvate à Ethanol Adenosine à Inosine
DECARBOXYLASE DEAMINASE

ALANINE
Pyruvate à Alanine
AMINOTRANSFERASE ACITIVATORS AND INHIBITORS
DOPA Inhibits pyruvate dehydrogenase by
DOPA à Dopamine ARSENIC
DECARBOXYLASE binding to lipoic acid

DOPAMINE β-
Dopamine à Norepinephrine Inhibitor of aconitase FLUOROACETATE
HYDROXYLASE
PHENYL- FRUCTOSE-2,6-
ETHANOLAMINE- Activator of PFK-1
BISPHOSPHATE
Norepinephrine à Epinephrine N-METHYL-
TRANSFERASE N-
Activator of CPS-I
(PNMT) ACETYLGLUTAMATE
GLUTAMATE
Glutamate à GABA
DECARBOXYLASE GLUCOKINASE VS. HEXOKINASE
HISTIDINE Present in liver parenchymal cells
Histidine à Histamine GLUCOKINASE
DECARBOXYLASE and islet cells of the pancreas
PHENYLALANINE Phosphorylates glucose and other
Phenylalanine à Tyrosine BOTH
HYDROXYLASE hexoses

TOPNOTCH MEDICAL BOARD PREP BIOCHEMISTRY PEARLS DIGITAL HANDOUT BY DRS. RUBIO AND BATICULON Page 45 of 49
For inquiries visit www.topnotchboardprep.com.ph or email us at topnotchmedicalboardprep@gmail.com
This handout is only valid for the March 2023 PLE batch. This will be rendered obsolete for the next batch since we update our handouts regularly.
TOPNOTCH MEDICAL BOARD PREP BIOCHEMISTRY PEARLS DIGITAL HANDOUT BY DRS. RUBIO AND BATICULON
For inquiries visit www.topnotchboardprep.com.ph or https://www.facebook.com/topnotchmedicalboardprep/
This handout is only valid for the March 2023 PLE batch. This will be rendered obsolete for the next batch since we update our handouts regularly.
Functions as a glucose sensor in the Building block of fatty acids and
maintenance of blood glucose GLUCOKINASE ACETYL COA
cholesterol
homeostasis
Main substrate used for GLUCOGENIC AMINO
Low Km HEXOKINASE gluconeogenesis during a fast ACIDS

Intermediate that can never be used


High Vmax GLUCOKINASE ACETYL COA
for gluconeogenesis
During the first 18 to 24 hours of
Phosphorylates glucose to meet the starvation, which pathway is
HEXOKINASE
cell’s needs responsible in maintaining blood GLYCOGENOLYSIS
Phosphorylates glucose for storage glucose levels?
GLUCOKINASE
as glycogen
After 24 hours of starvation, which
pathway is mainly responsible for GLUCONEOGENESIS
Activity induced by insultin GLUCOKINASE
maintaining blood glucose levels?
Percentage of gluconeogenesis that
occurs in the kidney during a 40%
NADH OR NADPH? prolonged fast
Hours of fasting before glycogen
Pyruvate à Lactate NADH 18 hours
stores are exhausted
CATABOLISM OF
Acetyl CoA à Palmitate NADPH Cause of death in prolonged
ESSENTIAL AMINO
starvation
ACIDS
GLUCAGON >>
NADH, FADH2, GTP, CO2? INSULIN
#GLUCONEOGENESIS
Succinate thiokinase GTP Metabolic effects of uncontrolled FROM AMINO ACIDS,
diabetes mellitus #LIPOLYSIS IN
α-ketoglutarate dehydrogenase NADH, CO2 ADIPOSE TISSUE,
#KETOGENESIS IN
Malate dehydrogenase NADH LIVER
HYPOGLYCEMIA
Metabolic effects of acute alcohol
LACTIC ACIDOSIS
Isocitrate dehydrogenase NADH, CO2 intoxication
KETOACIDOSIS
Massive increase of this molecule
Aconitase NONE OF THE ABOVE leads to metabolic effects of alcohol NADH
intoxication
Succinate dehydrogenase FADH2 Intermediate in alcohol metabolism
that causes flushing, tachycardia,
ACETALDEHYDE
hyperventilation, and
INTEGRATION OF METABOLISM nausea
Predominant hormone in the well DIABETIC
INSULIN Ketoacidosis with hyperglycemia
fed state KETOACIDOSIS

Hormone that helps maintain blood PROLONGED


GLUCAGON STARVATION
glucose levels during starvation Ketoacidosis with hypoglycemia
ALCOHOL
EPINEPHRINE, INTOXICATION
Exert effects similar to glucagon CORTISOL, GROWTH Critical intermediate necessary to
HORMONE maintain TCA cycle activity usually
OXALOACETATE
Ratio of carbon dioxide RESPIRATORY depleted in prolonged starvation
produced/oxygen consumed QUOTIENT and alcohol intake
TCA intermediate that is depleted
Main metabolic fuel of neurons and α-KETOGLUTARATE
GLUCOSE during hyperammonemia
erythrocytes
TCA intermediate that links it to the
FUMARATE
Main metabolic fuel of heart muscle FATTY ACIDS urea cycle

Main fuel of the skeletal and heart WEIGHT IN KG


Formula for BMI computation
muscle, and alternative fuel of the KETONE BODIES HEIGHT IN M2
brain during a fast On ketogenic diet, these pathways
Percentage of the brain’s energy GLUCONEOGENESIS
that remain active in the liver even
requirement that can be met by 20% KETOGENESIS
during the fed state
ketone bodies
Product of aerobic glycolysis PYRUVATE NITROGEN METABOLISM
State of nitrogen balance in
POSITIVE
Product of anaerobic glycolysis LACTATE pregnancy

State of nitrogen balance in


NEGATIVE
Product of glycogenolysis in liver GLUCOSE malignancy
Which 3 compounds are the
GLUCOSE-6- immediate donors of the atoms of ASPARTATE, NH3, CO2
Product of glycogenolysis in muscle urea?
PHOSPHATE

TOPNOTCH MEDICAL BOARD PREP BIOCHEMISTRY PEARLS DIGITAL HANDOUT BY DRS. RUBIO AND BATICULON Page 46 of 49
For inquiries visit www.topnotchboardprep.com.ph or email us at topnotchmedicalboardprep@gmail.com
This handout is only valid for the March 2023 PLE batch. This will be rendered obsolete for the next batch since we update our handouts regularly.
TOPNOTCH MEDICAL BOARD PREP BIOCHEMISTRY PEARLS DIGITAL HANDOUT BY DRS. RUBIO AND BATICULON
For inquiries visit www.topnotchboardprep.com.ph or https://www.facebook.com/topnotchmedicalboardprep/
This handout is only valid for the March 2023 PLE batch. This will be rendered obsolete for the next batch since we update our handouts regularly.

α-ketoacid of alanine PYRUVATE Really a hormone VITAMIN D

Lipid soluble antioxidant with no


α-ketoacid glutamate α-KETOGLUTARATE precisely defined metabolic VITAMIN E
function
α-ketoacid aspartate OXALOACETATE Carrier of one carbon units or
FOLATE
methyl groups
Amino acid that performs a central
role in removal of nitrogen from GLUTAMATE DOPA decarboxylase VITAMIN B6
most other amino acids
Transports ammonia from muscle
ALANINE Dopamine hydroxylase VITAMIN C
to the liver
Transports ammonia from
peripheral tissues to the liver and GLUTAMINE Cystathionine synthase VITAMIN B6
kidney

Methionine synthase VITAMIN B12
ACTIVATED VS. INHIBITED
High insulin Ιntense yellow color used as food
INHIBITED VITAMIN B2
Glucose-6-phosphatase additive
Low glucagon
ACTIVATED
Pyruvate kinase Used to treat hyperlipidemia VITAMIN B3
High epinephrine
INHIBITED
HMG CoA reductase
High cAMP
ACTIVATED
VITAMIN DEFICIENCIES
Hormone sensitive lipase
Phosphorylation Dermatitis, cheilosis, glossitis VITAMIN B2
INHIBITED
Glycogen synthase
Dephosphorylation Burning foot syndrome VITAMIN B5
ACTIVATED
Pyruvate dehydrogenase

IDENTIFY THE VITAMIN Rickets and osteomalacia VITAMIN D

Transketolase reactions in the HMP


VITAMIN B1 Hemolysis VITAMIN E
Shunt

Prevents atherosclerotic plaque Leading cause of preventable


VITAMIN E VITAMIN A
formation blindness
Normalizes calcium levels in Hemorrhagic disease of the
VITAMIN D VITAMIN K
response to hypocalcemia newborn

Acetyl Coa to Malonyl CoA BIOTIN Subacute combined degeneration VITAMIN B12

Hydroxylation of collagen VITAMIN C FOLATE and VITAMIN


Megaloblastic anemia
B12
Deficiency results from Hartnup
VITAMIN B3 Pernicious anemia VITAMIN B12
disease and carcinoid syndrome

Condensation of glycine and


VITAMIN B6 Anencephaly and spina bifida FOLATE
succinyl CoA

All aminotransferases VITAMIN B6 Pellagra VITAMIN B3

Carboxylation of glutamic acid


VITAMIN K Wernicke-Korsakoff syndrome VITAMIN B1
residues in coagulation factors

Co-factor for acyl transfers and


VITAMIN B5 Scurvy VITAMIN C
component of fatty acid synthase

Glycogen phosphorylase VITAMIN B6 Beriberi VITAMIN B1

Increases iron absorption VITAMIN C Methylmalonic aciduria VITAMIN B12

Conversion of propionyl CoA to BIOTIN and VITAMIN


glucose B12 VITAMIN TOXICITIES
Not strictly a vitamin because it can
be synthesized in the body from an VITAMIN B3 Teratogenic VITAMIN A
amino acid
Required to synthesize niacin from Pseudotumor cerebri VITAMIN A
VITAMIN B6
tryptophan

TOPNOTCH MEDICAL BOARD PREP BIOCHEMISTRY PEARLS DIGITAL HANDOUT BY DRS. RUBIO AND BATICULON Page 47 of 49
For inquiries visit www.topnotchboardprep.com.ph or email us at topnotchmedicalboardprep@gmail.com
This handout is only valid for the March 2023 PLE batch. This will be rendered obsolete for the next batch since we update our handouts regularly.
TOPNOTCH MEDICAL BOARD PREP BIOCHEMISTRY PEARLS DIGITAL HANDOUT BY DRS. RUBIO AND BATICULON
For inquiries visit www.topnotchboardprep.com.ph or https://www.facebook.com/topnotchmedicalboardprep/
This handout is only valid for the March 2023 PLE batch. This will be rendered obsolete for the next batch since we update our handouts regularly.

Hypercalcemia and calcinosis VITAMIN D


DNA VS. RNA
Chargaff’s Rules DNA
Sensory neuropathy VITAMIN B6
Contains uracil RNA
Dilation of blood vessels, flushing,
VITAMIN B3
liver damage
Usually double stranded DNA
Least toxic of fat-soluble vitamins VITAMIN E
More susceptible to hydrolysis RNA
Most toxic of water-soluble vitamins VITAMIN B6

CENTRAL DOGMA
QUESTIONS ON VITAMINS
LOSS OF SENSITIVITY DNA to RNA TRANSCRIPTION
TO GREEN LIGHT à
Signs of vitamin A deficiency IMPAIRMENT TO
RNA to Protein TRANSLATION
ADAPT TO DIM LIGHT
à NIGHT BLINDNESS
DNA to DNA REPLICATION
Major source of vitamin D SYNTHESIS IN SKIN

CALCIDIOL (25- REVERSE


RNA to DNA
HYDROXY TRANSCRIPTION
Main storage form of vitamin D CHOLECALCIFEROL) TRANSCRIPTION,
BOUND TO VITAMIN D Pribnow Box
Prokaryotic
BINDING GLOBULIN
25 HYDROXYLATION: TRANSCRPTION,
Organs responsible for LIVER Hogness Box, CAAT Box
Eukaryotic
hydroxylation of vitamin d 1 HYDROXYLATION:
KIDNEY Shine Dalgarno sequence TRANSLATION
REDUCES LP(A) AND
Effect of niacin on lipoproteins
INCREASES HDL
Lac Operon TRANSCRIPTION
Diet associated with pellagra CORN-BASED DIET
Rho and Sigma factor TRANSCRIPTION
B12 binding protein secreted in the
COBALOPHILIN
saliva DNA strand copied during RNA
TEMPLATE STRAND
Glycoprotein secreted by parietal synthesis
cells necessary for intestinal INTRINSIC FACTOR
DNA strand matching the sequence
absorption of B12 CODING STRAND
of the RNA transcript
Vitamin B12
deficiency leads to impairment of Short T-G repeats at end of each
FOLATE TRAP chromosome responsible for aging TELOMERES
methionine synthase, resulting in
(FUNCTIONAL or cell survivability and malignancy
accumulation of homocysteine and
FOLATE DEFICIENCY)
trapping of folate as
methyltetrahydrofolate Coding regions of DNA EXONS
Present in raw egg whites, inhibits
AVIDIN
absorption of biotin Noncoding regions of DNA INTRONS
Coagulation system proteins FACTORS 2,7,9,10
affected by vitamin K PROTEIN C and S
IDENTIFY THE ENZYME/PROTEIN
OSTEOCALCIN AND
Calcium-binding proteins that Unwinds double helix HELICASE
MATRIX GLA
require vitamin K
PROTEIN Removes supercoils TOPOISOMERASE
ORIGIN RECOGNITION
IDENTIFY THE MINERAL Identifies origin of replication COMPLEX (DNAa
PROTEIN)
Anemia and hemochromatosis IRON Synthesis of RNA primer PRIMASE
SINGLE-STRANDED
Maintains the separation of the
DNA-BINDING
parental strands
Glutathione peroxidase SELENIUM PROTEINS
Elongation of leading strand in the
Carbonic anhydrase, alcohol DNA POLYMERASE III
5’ to 3’ direction
dehydrogenase, DNA and RNA ZINC Excision of primers DNA POLYMERASE I
polymerase Seals the nick between Okazaki
Cytochrome oxidase, dopamine β- LIGASE
fragments
hydroxylase, monoamine oxidase, COPPER Synthesis of rRNA RNA POLYMERASE I
tyrosinase
Synthesis of mRNA RNA POLYMERASE II
Thyroid hormone synthesis IODINE Synthesis of tRNA RNA POLYMERASE III

Inorganic substance of bones and
FLUORINE
teeth

TOPNOTCH MEDICAL BOARD PREP BIOCHEMISTRY PEARLS DIGITAL HANDOUT BY DRS. RUBIO AND BATICULON Page 48 of 49
For inquiries visit www.topnotchboardprep.com.ph or email us at topnotchmedicalboardprep@gmail.com
This handout is only valid for the March 2023 PLE batch. This will be rendered obsolete for the next batch since we update our handouts regularly.
TOPNOTCH MEDICAL BOARD PREP BIOCHEMISTRY PEARLS DIGITAL HANDOUT BY DRS. RUBIO AND BATICULON
For inquiries visit www.topnotchboardprep.com.ph or https://www.facebook.com/topnotchmedicalboardprep/
This handout is only valid for the March 2023 PLE batch. This will be rendered obsolete for the next batch since we update our handouts regularly.

mRNA, tRNA, rRNA, snRNA Phospholipid with a sphingosine


SPHINGOPHOSPHOLIPID
backbone
Largest mRNA
No serial repeats, relatively short
GLYCOPROTEINS
Smallest tRNA carbohydrate chains

May contain glucuronic acid and


Most abundant rRNA GLYCOSAMINOGLYCANS
iduronic acid
Contained in the machinery for May contain N-
rRNA GLYCOSAMINOGLYCANS
protein synthesis acetylglucosamine and N-
GLYCOPROTEINS
Carries genetic information from acetylgalactosamine
nuclear DNA to the cytosol, serves May contain choline,
mRNA PHOSPHOLIPIDS
as the template for protein ethanolamine, serine, inositol
synthesis
Include cerebrosides, globosides,
Contains abundant unusual bases tRNA GLYCOLIPIDS
gangliosides, sulfatides
Include hyaluronic acid,
Poly-A tail mRNA
chondroitin sulfate, keratin
GLYCOSAMINOGLYCANS
sulfate, heparin, heparin sulfate,
3’-CCA sequence tRNA dermatan sulfate
Cloverleaf shape and serves as the Include mucins, TSH,
tRNA GLYCOPROTEINS
adapter molecule during translation immunoglobulins

Methylguanosine cap mRNA


IDENTIFY THE COMPLEX MOLECULE
Contains the codon mRNA Major component of lung surfactant, DIPALMITOYL-
deficient in hyaline membrane PHOSPHATIDYL-
Contains the anticodon tRNA disease CHOLINE

mRNA processing and gene Only glycerophospholipid that is


snRNA CARDIOLIPIN
regulation antigenic
Reservoir for arachidonic acid in the
PHOSPHATIDYL-
membranes and precursor for IP3
GENETIC CODE and DAG
INOSITOL
Conserved from very early
UNIVERSAL Important constituent of myelin SPHINGOMYELIN
stages of evolution

Given amino acid may have more


DEGENERATE Important anticoagulant HEPARIN
than one codon coding for it

Codon always code for the same SPECIFIC/UNAMBIGUOUS Attracts water into the ECM for cell
amino acid HYALURONIC ACID
migration, wound repair
NONOVERLAPPING, Determines charge selectiveness of
No punctuation between codons HEPARAN SULFATE
COMMALESS renal glomerulus

Located at sites of calcification of CHONDROITIN
IDENTIFY THE RIBOSOMAL BINDING SITE endochondral bone SULFATE

Empty tRNA E SITE Main GAG of the skin DERMATAN SULFATE

Growing peptide chain P SITE Maintains transparency of the


KERATAN SULFATE
cornea

Incoming aminoacyl tRNA A SITE Lubricant and protective agent MUCIN

Give the 3 stop codons UGA, UAA, UAG CERULOPLASMIN


Transport molecules
TRANSFERRIN

Give the codon for methionine AUG Immunologic molecules IMMUNOGLOBULINS



N-ACETYL-
COMPLEX MOLECULES Oligosaccharide in blood type A
GALACTOSAMINE
Heteropolysaccharides
consisting of repeating units of GLYCOSAMINOGLYCANS Oligosaccharide in blood type B GALACTOSE
amino sugars and acidic sugars
Proteins to which
GLYCOPROTEINS
oligosaccharides are attached

Core protein to which


PROTEOGLYCANS
glycosaminoglycans are attached

Derivatives of ceramide SPHINGOSINE

Alcohol attached by a
phosphodiester bridge to PHOSPHOLIPID
diacylglycerol
TOPNOTCH MEDICAL BOARD PREP BIOCHEMISTRY PEARLS DIGITAL HANDOUT BY DRS. RUBIO AND BATICULON Page 49 of 49
For inquiries visit www.topnotchboardprep.com.ph or email us at topnotchmedicalboardprep@gmail.com
This handout is only valid for the March 2023 PLE batch. This will be rendered obsolete for the next batch since we update our handouts regularly.

You might also like